You are on page 1of 72

Acknowledgements and Important Notice:

4B
All questions from the HKDSE and HKCEE are reproduced by permission of the HKEAA. All questions from the contests of University
of Waterloo are reproduced by permission of the Centre for Education in Mathematics and Computing, University of Waterloo. The

8
Australian Mathematics Competition problems are printed in this publication with permission from the Australian Mathematics Trust. Number and Algebra
© Australian Mathematics Trust. Unauthorized use of the aforementioned questions in this electronic version is prohibited.

Logarithmic Functions
✂ 8.1 Common Logarithms (P. 8.2)
✂ 8.2 Logarithms with an Arbitrary Base (P. 8.12)
✂ 8.3 Logarithmic Functions and Their Graphs (P. 8.20)
✂ 8.4 Exponential Equations and
Logarithmic Equations (P. 8.31)
✂ 8.5 Applications of Logarithms (P. 8.37)
✂ 8.6 History of Logarithm Tables and Slide Rules (P. 8.49)

Great Earthquakes in Japan


In the 2010s, two great earthquakes caused severe damage in Japan:
Year Location Magnitude on the Richter Scale
2011 East Japan 9.0
2016 Kumamoto 7.0

The calculation of magnitudes on the Richter Scale is an application


of logarithmic functions.

Q& A
When two earthquakes differ by x in magnitude, the energy released in the
1.5x
greater one is 10 times in the other. How many times the energy released
in a 5.0-magnitude earthquake is the energy released in a 6.0-magnitude
one? (Give the answer correct to the nearest integer.)
8. 2 | Chapter 8

8.1

Common Logarithms
Non-foundation

A Definition of Common Logarithms


We know that
1 2 3
10 = 10 , 100 = 10 and 1 000 = 10 ,
i.e. 10, 100 and 1 000 can all be expressed as powers of 10.
y
In fact, from the graph of x = 10 , we know that other positive numbers
(such as 2, 4 and 0.5) can also be expressed as powers of 10.
x
5 From the graph, we have
0.3
2 . 10
x = 10 y
4 4 . 100.6
0.5 . 10-0.3
3

0.5
y
– 0.3 – 0.2 – 0.1 0 0.1 0.2 0.3 0.4 0.5 0.6 0.7 0.8

When a positive number x is expressed in the form 10y, the index y is


called the logarithm of x with base 10, or the common logarithm of x. We
can denote y by log x.

If x = 10y, then y = log x.

e.g. (i) a 100 = 102 (ii) a 0.01 = 10


-2

` log 100 = 2 ` log 0.01 = -2

The converse of the above relation is also true.

If y = log x, then x = 10y.

Note: For x G 0, log x is undefined.


e.g. log 0 and log (-3) are undefined.

logarithm 對數 common logarithm 常用對數

E_SrSecMath_4B08.indd 2 22/5/2023 下午2:44


Logarithmic Functions | 8. 3

y
We can perform the interconversion between x = 10 and log x = y. ✂

Non-foundation
e.g. (i) 100 = 102 log 100 = 2

-1
(ii) 0.1 = 10 log 0.1 = -1

Quick Quiz 8.1

Complete the following table.

x = 10y log x = y
4
1. 10 000 = 10 log 10 000 =
( )
2. 1 000 000 = 10 log 1 000 000 = 6
-2
3. 0.01 = 10 log 0.01 =
0.001 = 10(
)
4. log 0.001 = -3
5. c = 102.5 log c =
6. 45 = 10( )
log 45 = d

[Find the values of common logarithms by the definition]

Example Find the values of the following logarithms.


Level 1
1
(a) log 1 000 (b) log 0.000 1 (c) log 1
100

3
Solution (a) a 1 000 = 10 ◀ Express 1 000 as a power of 10.
` log 1 000 = 3 ◀ If x = 10y, then log x = y.

-4
(b) a 0.000 1 = 10 ◀ Move the decimal point 4 places to the right.
0.000 1 = 10-4
` log 0.000 1 = - 4

(c) a 1 = 1 = 10-2
100 10
2

` log 1 = - 2
100

Instant Drill 1
Find the values of the following logarithms.
(a) log 100 000 (b) log 0.000 001 (c) log 1
10 000
➥ Ex 8A 8
8.4 | Chapter 8

✂ We can also use the log key on a calculator to find the values of common Quick Quiz 8.2
logarithms.
Non-foundation

Use a calculator to find


e.g. Common logarithm Keying sequence Display the values of the following
(i) log 76 log 76 ) EXE 1.8808g logarithms, correct to
3 significant figures.
(ii) log 7 log 7 a b/c 9 ) EXE -0.10914g
9 1. log 189 2. log 18
5
3. log 0.27

[Find the unknowns in common logarithms by the definition]

Example Find the value of x in each of the following.


Level 1
2
(Give the answers correct to 3 significant figures if necessary.)
(a) log x = 2 (b) log x = 1.5 (c) log x = -0.4

Solution (a) a log x = 2


` x = 102
= 100

(b) a log x = 1.5


Calculator
` x = 101.5 Keying sequences:
= 31.6, cor. to 3 sig. fig. (b) 10 ^ 1.5 ) EXE

[31.622…]
(c) a log x = -0.4
(c) 10 ^ – 0.4 ) EXE
` x = 10-0.4 [0.39810…]
= 0.398, cor. to 3 sig. fig.

Instant Drill 2
Find the value of x in each of the following.
(Give the answers correct to 3 significant figures if necessary.)
(a) log x = 0 (b) log x = 4
(c) log x = 1.7 (d) log x = -0.6
➥ Ex 8A 9–11

B Properties of Common Logarithms


According to the definition, if x = 10y, then y = log x. Substituting x = 10y
into y = log x, we have

log 10y = y

Hence, log 10 = 1 and log 1 = 0 ◀


0
10 = 1
Logarithmic Functions | 8. 5

Let’s investigate more properties of common logarithms. ✂

Non-foundation
Class Activity 8.1
Objective: To investigate the properties of common logarithms.

[Use a calculator in this activity. Give the answers correct to 3 significant figures if necessary.]

1. (a) Complete the following table.

M N log MN log M # log N log M + log N


10 10 log (10 # 10) = log 10 # log 10 = log 10 + log 10 =
2 3 log (2 # 3) = log 2 # log 3 = log 2 + log 3 =

(b) According to the results of (a), do you think each of the following is correct?

(i) log MN = log M # log N Yes No

(ii) log MN = log M + log N Yes No

2. (a) Complete the following table.

log M
M N log M log M - log N
N log N
log 10
10 10 log 10 = = log 10 - log 10 =
10 log 10
log 10
10 2 log 10 = = log 10 - log 2 =
2 log 2

(b) According to the results of (a), do you think each of the following is correct?
log M
(i) log M = Yes No
N log N

(ii) log M = log M - log N Yes No


N

3. (a) Complete the following table.

M k log Mk (log M)k k log M


2 2
10 2 log 10 = (log 10) = 2 log 10 =
5 2 log 52 = (log 5)2 = 2 log 5 =

(b) According to the results of (a), do you think each of the following is correct?
k k
(i) log M = (log M) Yes No
k
(ii) log M = k log M Yes No
8. 6 | Chapter 8

✂ From Class Activity 8.1, we can observe the following properties of


common logarithms:
Non-foundation

For any positive numbers M and N, Do’s and Don’ts


1. log MN = log M + log N In general,
1. log M + log N ! log (M + N)
2. log M = log M - log N
N 2. log M - log N ! log (M - N)
3. log M k = k log M, where k is a real number

Proof
For positive numbers M and N, let r = log M and s = log N.
r s y
Then M = 10 and N = 10 . ◀ If y = log x, then x = 10 .

r s
1. MN = 10 :10
MN = 10r + s ◀ a p:a q = a p + q

log MN = r + s ◀ If x = 10y, then log x = y.

` log MN = log M + log N


r
2. M = 10
N s
10
p
M = 10r - s ◀ a p-q
q = a
N a

log M = r - s ◀
y
If x = 10 , then log x = y.
N

` log M = log M - log N


N

3. M k = (10r)k
M k = 10kr ◀
p q
(a ) = a
pq

log M k = kr ◀
y
If x = 10 , then log x = y.
` log M k = k log M

Quick Quiz 8.3

Determine whether each of the following is correct. Put ‘✓’ in if it is correct, and ‘✗’ if it is wrong.

1. log (1 + 4) = log 1 + log 4 2. log 8 + log 4 = log (8 # 4)

3. log (9 ' 7) = log 9 ' log 7 4. log (23 ' 5) = log 23 - log 5
log 15
5. log 11 - log 13 = log 11 6. = log 5
13 log 3
4 4 5
7. (log 5) = log 5 8. log 7 = 5 log 7
Logarithmic Functions | 8.7

[Use the properties of common logarithms to find the values of expressions]


Non-foundation
Example Without using a calculator, find the values of the following.
Level 1
3
(a) log 2 + log 5
(b) log 300 - log 3
log125
(c)
log25

Solution (a) log 2 + log 5 = log (2 # 5) ◀ log M + log N = log MN

= log 10
= 1 ◀ 10 = 101 and log 10y = y

(b) log 300 - log 3 = log 300 ◀ log M - log N = log M


3 N

= log 100
= log 102
=2 ◀ log 10y = y

(c) Do’s and Don’ts


log 125
Note that ! 125 .
log 25 25
We should express 125 and 25 as powers of 5.

3
log125 log 5
= 2
log 25 log 5

3 log 5 k
= ◀ log M = k log M
2 log 5
= 3
2

Instant Drill 3
Without using a calculator, find the values of the following.
(a) log 4 + log 25 (b) log 125 + log 8
(c) log 50 - log 5 (d) log 9 - log 90
log16 log 27
(e) (f)
log8 log 81
➥ Ex 8A 12–17

E_SrSecMath_4B08.indd 7 22/5/2023 下午2:44


8. 8 | Chapter 8

✂ [Simplify expressions with common logarithms]

Simplify the following expressions, where x 2 0 and x ! 1.


Non-foundation

Example
Level 2
4 3 -4
(a) log x + log x

log x - log 1x
2
(b) 3
log x

Solution Key
Use log x to express the terms that are in the form log xk.
k
log x k log x

3 -4
(a) log x + log x = 3 log x + (-4) log x ◀ log M k = k log M
= (3 - 4) log x
= - log x

Alternative
3 -4 3 -4
log x + log x = log (x :x ) ◀ log M + log N = log MN
3 + (-4)
= log x
= log x-1
= (-1) log x ◀ log M k = k log M
= - log x

log x - log 1x
2 2 1
log x - log x- Rewrite them in the
(b) =
3
log x
1 form x k first
log x 3
2 log x - (-1) log x
= ◀ log M k = k log M
1 log x
3
3 log x
=
1 log x
3
=9

Instant Drill 4
Simplify the following expressions, where x 2 0 and x ! 1.
3
log x
(b) log x + log 12
5 -3 4
(a) log x - log x (c)
x log x - log x
➥ Ex 8A 38–45

E_SrSecMath_4B08.indd 8 22/5/2023 下午2:44


Logarithmic Functions | 8. 9

[Use variables to express common logarithms]


Non-foundation
Example Let log 2 = a and log 3 = b. Express the following logarithms
Level 2
5
in terms of a and b.
(a) log 18 (b) log 0.6 (c) log 15

2
Solution (a) log 18 = log (2 # 3 )
2 Key
= log 2 + log 3 ◀ log MN = log M + log N
Rewrite the number in
= log 2 + 2 log 3 ◀ log M k = k log M
each question as a result of
= a + 2b multiplying/dividing 2, 3 and 10.
2
(a) 18 = 2 # 9 = 2 # 3
(b) log 0.6 = log 2 # 3 (b) 0.6 = 6 = 2 # 3
10 10 10
= log 2 + log 3 - log 10
(c) 15 = 3 # 5 = 3 # 10
=a+b-1 ◀ log 10 = 1 2

(c) log 15 = log `3 # 10 j


2
= log 3 + log 10 - log 2 Exam Video
=b+1-a (for DSE Level 4+)

Instant Drill 5
Let log 2 = a and log 7 = b. Express the following logarithms in terms
of a and b.

(a) log 14 (b) log 4


7
➥ Ex 8A 46–49
(c) log 280 (d) log 0.35
★ Public Exam Question 13

Class Practice 8.1


1. If log x = -1, find the value of x.

Without using a calculator, find the values of the following. [Nos. 2–4]
log 49
2. log 2 + log 50 3. log 8 - log 80 4.
log 7
Simplify the following expressions, where x 2 0 and x ! 1. [Nos. 5–6]
-5 log x
5. log x + log x4 6. 2
log x
Exam-type
7. Let log 3 = a and log 4 = b. Express the following logarithms in
terms of a and b.

(a) log 12 (b) log 4 (c) log 120


3
8.10 | Chapter 8

✂ Exercise 8A
Non-foundation

Level 1

1. Use a calculator to find the values of the following logarithms. Formulae Station

(Give the answers correct to 3 significant figures.) • If x = 10y, then y = log x.


• If y = log x, then x = 10y.
(a) log 24 (b) log 3 000 (c) log 0.05
• log 10y = y
• log 10 = 1
Convert the following expressions into the form log x = y. [Nos. 2–4]
• log 1 = 0
5
2. 10 = 100 000 3. 10 -2
= 1 4.
n
10 = 0.2 • log MN = log M + log N
100
• log M = log M - log N
y N
Convert the following expressions into the form x = 10 . [Nos. 5–7]
• log M k = k log M
5. log 1 000 = 3 6. log 0.000 01 = -5 7. log A = 0.9

8. Without using a calculator, find the values of the following logarithms.

(a) log 10 000 000 ➥ Example 1 (b) log 0.1 (c) log 1
1 000

Find the value of x in each of the following. [Nos. 9–11]


(Give the answers correct to 3 significant figures if necessary.)
9. log x = 1 ➥ Example 2 10. log x = 0.8 11. log x = -1.06

Without using a calculator, find the values of the following. [Nos. 12–17]
12. log 4 + log 250 ➥ Example 3(a) 13. log 2 + log 0.5

14. log 6 - log 60 ➥ Example 3(b) 15. log 5 - log 1


20

log 81 log 7
16. ➥ Example 3(c) 17.
log 9 log 49

Simplify the following expressions, where x 2 0 and x ! 1. [Nos. 18–23]


3
log x log x 5
18. 19. 20. log x - 5 log x
log x log x

21. log 1x + 2 log x 22. log (1 000x) - log x 23. log 10


x + log x

24. If log 2 = a, express the following logarithms in terms of a.

(a) log 4 (b) log 1 (c) log 2


2
(d) log 20 (e) log 5

Exam-type
25. If log 2 = m and log 3 = n, express the following logarithms in terms of m and n.

(a) log 2 (b) log 6 (c) log 12 (d) log 9


3 8
Logarithmic Functions | 8.11

Level 2 ✂

Non-foundation
Without using a calculator, find the values of the following logarithms. [Nos. 26–28]
5 3 -2
26. log 100 27. log 10004 28. log 0.1

Without using a calculator, find the values of the following. [Nos. 29–37]
4 1
29. log 20 - log 2 30. log (2 :5 3 ) - log 5 3 31. log 25 + 2 log 2

log 3
32. log 15 + log 8 - log 12 33. 3 log 4 + 2 log 5 - log 16 34.
log 9 3

log 14 - log 2 log 2 3 log 1 + 3 log 1


35. 36. 37. 8 5
log 49 log 3 + 2 log 2 log 100

Simplify the following expressions, where x 2 0 and x ! 1. [Nos. 38–45]


6
log x - 2 log x log (10x) - 1 log x + log 5
38. 3 ➥ Example 4 39. 40. 4
log x log x log (5x)
1 1
3 3
log x 2 + 2 log x 2 log x log x
41. 42. 43.
log x 3 log x - log x
2
log x

log x 3 log x + 1 log x


44. 45. 2
2
log x + log x 2 log x

Exam-type
46. If log 2 = p and log 3 = q, express the following logarithms in terms of p and q.
3
(a) log 24 (b) log 60 (c) log 2 (d) log 27
➥ Example 5
9 2

Exam-type
47. If log 3 = a and log 5 = b, express the following logarithms in terms of a and b.

(a) log 45 (b) log 135 (c) log 75 (d) log 2


81
Exam-type
48. If log x = a and log y = b, express the following in terms of a and b, where x 2 0, y 2 0 and
x ! 1, y ! 1.

(b) log x (d) log 3 xy


2
(a) log (10xy) (c) log (x y)
y
Exam-type p q
49. It is given that 4 = 10 and 6 = 10 . Express log 9 in terms of p and q.

Level 3
n
50. If log [log (log 10 )] = 1, find the value of n.
(Give the answer in scientific notation.)

51. (a) It is given that A is an integer such that log A = 2.7 (cor. to 1 d.p.). Without using a calculator,
show that A is a three-digit number.

(b) Logarithm log 24 log 365 log 2 023


Value (cor. to 3 d.p.) 1.380 2.562 3.306
Using the table, show that the result of 24 # 365 # 2 023 is an eight-digit number.

E_SrSecMath_4B08.indd 11 22/5/2023 下午2:44


8.12 | Chapter 8

8.2

Logarithms with an Arbitrary Base
Non-foundation

A Definition of Logarithms with an Arbitrary Base


In the previous section, we learnt that if x = 10y, then:
x = 10y y = log x is the logarithm of x with base 10,
i.e. the common logarithm.
y
Similarly, if x = 2 , then:
x = 2y y = log2 x is the logarithm of x with base 2.

y
In general, if x = a (where a 2 0 and a ! 1), then y is called the
logarithm of x with base a, denoted by y = loga x.

If x = ay, then y = loga x, where a 2 0 and a ! 1. ◀ Consider a = 10. We have


the definition of the common
logarithm:
e.g. (i) a 9 = 32 (ii) a 1 = 4–1
If x = 10y, then y = log10 x, where
4
log10 x is usually written as log x.
` log3 9 = 2 ` log4 1 = –1
4

The converse of the above relation is also true.

If y = loga x, then x = ay, where a 2 0 and a ! 1.

Note: Only when a 2 0, a ! 1 and x 2 0, loga x is defined.


For x G 0, loga x is undefined.
e.g. log-3 5, log1 7 and log9 (-3) are undefined.

Quick Quiz 8.4

Complete the following tables.

x = ay loga x = y x = ay loga x = y
x = ay loga x = y
4 base
1. 16 = 2 5. log7 1 = 0

2. 36 = 62 6. log9 81 = 2

3. 1 = 5-2 7. 1 = -1
log 10
25

4. 12 = 10m 8. log2 n = 1.5


Logarithmic Functions | 8.13

[Find the values of logarithms by the definition]


Non-foundation
Example Find the values of the following logarithms.
Level 1
6
(a) log2 8 (b) log5 1 (c) log3 3
25

3
Solution (a) a 8 = 2 ◀ Since the base is 2, express 8 as a power of 2.
` log2 8 = 3 ◀ If x = ay, then loga x = y.

(b) a 1 = 1 = 5-2 ◀ Express 1 as a power of 5.


25 5
2 25

` log5 1 = - 2
25
1
(c) a 3 = 32 ◀ Express 3 as a power of 3.

` log3 3 = 1
2

Instant Drill 6
Find the values of the following logarithms.
(a) log6 6 (b) log4 16 (c) log3 1
81
3
(d) log7 1 (e) log 1 7 (f) log5 5
7
➥ Ex 8B 2–7

We can also use the log key on a calculator to find the values of
logarithms with an arbitrary base.
e.g. Logarithm Keying sequence Display Quick Quiz 8.5
(i) log5 7 log 5 ’ 7 ) EXE 1.2090g Use a calculator to find
the values of the following
(ii) log3 4 log 3 ’ 4 a b/c 9 ) EXE -0.73814g
logarithms, correct to
9
3 decimal places.
(a) log4 9 (b) log7 6
(c) log 1 12
2

[Find the unknowns in logarithms by the definition]

Example Find the value of x in each of the following.


Level 1
7
(a) log 1 x = 3 (b) logx 49 = 2
2

Solution (a) a log 1 x = 3


2 Diagram Clue
x = `1j
3
` loga b = c
2

= 81
c
b=a
8.14 | Chapter 8

✂ (b) a logx 49 = 2
2
` x = 49
Non-foundation

x = 7 or -7 (rejected) ◀ The base of a logarithm cannot be negative.

Instant Drill 7
Find the value of x in each of the following.
(a) log3 x = -1 (b) logx 16 = 4
➥ Ex 8B 8–11

B Properties of Logarithms with an Arbitrary Base


Let a 2 0 and a ! 1. According to the definition, if x = ay, then
y
y = loga x. Substituting x = a into y = loga x, we have

loga ay = y

Logarithms with an arbitrary base have more properties as follows:

Let a 2 0, b 2 0 and a ! 1, b ! 1. For any positive


numbers M and N,
1. loga 1 = 0
2. loga a = 1
3. loga MN = loga M + loga N ◀ Properties of common logarithms:
log MN = log M + log N
4. log a M = loga M - loga N log M = log M - log N
N N
k log M k = k log M
5. loga M = k loga M, where k is a real number
log b M
6. loga M = , which is called the formula for
log b a
change of base
Property 6 can be used
to rewrite a logarithm into
logarithms with another base.
e.g. (i) log6 1 = 0 log 5 7
e.g. log2 7 =
(ii) log8 8 = 1 log 5 2

(iii) log3 2 + log3 5 = log3 (2 # 5) = log3 10

(iv) log4 3 - log4 7 = log4 3


7
3
(v) log5 2 = 3 log5 2

log 4
(vi) log3 4 =
log 3

E_SrSecMath_4B08.indd 14 22/5/2023 下午2:44


Logarithmic Functions | 8.15

Here, the proofs of Properties 1, 2 and 6 are shown. The proofs of ✂


Properties 3, 4 and 5 are similar to those of the common logarithms, and

Non-foundation
they are left to students as an exercise.

Proof
0 1
Properties 1 and 2 a a = 1 and a = a
` loga1 = 0 and loga a = 1
Property 6 Let loga M = p.
p
Then M = a .
p
logb M = logb a
= p logb a
= (loga M)(logb a)
log b M
` loga M =
log b a

[Use the properties of logarithms to find the values of expressions]

Example Without using a calculator, find the values of the following.


Level 1
8
(a) log2 6 - log2 48 (b) log6 4 + 2 log6 3 (c) log27 9

Solution (a) log2 6 - log2 48 = log2 6 ◀ loga M - loga N = log a M


48 N

= log2 1
8
1
◀ = 13 = 2-3
= log2 2-3 8 2
= -3 ◀ loga ay = y

2
(b) log6 4 + 2 log6 3 = log6 4 + log6 3 ◀ k loga M = loga Mk
2
= log6 (4 # 3 ) ◀ loga M + loga N = loga MN

= log6 36
= log6 62
=2

log 9 log b M
(c) log27 9 = ◀ Substitute b = 10 into loga M = and
log 27 log b a
2 write log10 as log.
log 3
= 3
log 3
Think
2 log 3
= If we change the base of log27 9
3 log 3
to another positive number
= 2 (e.g. 2 or 3), can we obtain the
3 same result?

E_SrSecMath_4B08.indd 15 22/5/2023 下午2:44


8.16 | Chapter 8

✂ Instant Drill 8
Non-foundation

Without using a calculator, find the values of the following.


(a) log6 3 + log6 2 (b) log5 50 - log5 2
(c) 3 log4 2 + log4 8 (d) log25 5
➥ Ex 8B 12–20

[Simplify expressions with logarithms]

Example Simplify the following expressions, where x 2 0, y 2 0


Level 2
9
and x ! 1, y ! 1.
log4 x
(a) (b) (logx y)(logy x)
log2 x

log x
log4 x log 4
Solution (a) =
log2 x log x
log 2
log x log 2
= 2 # 1
log 2 log x 2
log x log 2
= 2 log 2 # 1
2
log x
= 1

(b) (logx y)(logy x) = c mc m


log y log x
log x log y
1
= 1 ◀ Hence, logx y =
log y x
.

Instant Drill 9
Simplify the following expressions, where x 2 0, y 2 0 and x ! 1,
y ! 1.
log3 x3 2
(a) log9 x
(b) (logy x )(logx y)
➥ Ex 8B 40–44

Below illustrates another property of logarithms:


When loga M = loga N (where a, M and N are positive numbers, and a ! 1),
loga M - loga N = 0
log a M
N
=0
M =1 ◀ a0 = 1
N
M=N

E_SrSecMath_4B08.indd 16 22/5/2023 下午2:44


Logarithmic Functions | 8.17

Hence, we have the following property: ✂

Non-foundation
Let a, M and N be positive numbers, and a ! 1.
If loga M = loga N, then M = N.

We will use this property in the following example.

[Find the relation between two variables]

Example It is given that log9 y = log3 x, where x 2 0 and y 2 0.


Level 2
10
Express y in terms of x.

Solution log9 y = log3 x


log3 y log b M
= log3 x ◀ loga M =
log3 9 log b a
log3 y
2
= log3 x
log3 3
log3 y
= log3 x ◀ loga ay = y
2
log3 y = 2 log3 x Think
If we change the bases of log9 y
log3 y = log3 x2 Both sides have equal base.
and log3 x to 10, can we obtain
2
the same result?
y=x

Instant Drill 10
It is given that log2 x = log4 y, where x 2 0 and y 2 0. Express y in
terms of x.
➥ Ex 8B 45–48

Class Practice 8.2

1. Without using a calculator, find the value of log7 1 .


49
2. If log4 x = 2, find the value of x.

3. Without using a calculator, find the values of the following.


(a) log3 2 - log3 18 (b) log7 5 + log7 1
5
4. Simplify the following expressions, where x 2 0, y 2 0 and x ! 1,
y ! 1.
log2 x2 4
(a) (b) (log y x )(logx y)
log8 x

5. It is given that log5 y = log25 x . Express y in terms of x.


8.18 | Chapter 8

✂ Exercise 8B
Non-foundation

Level 1
1. Use a calculator to find the values of the following logarithms. Formulae Station
(Give the answers correct to 3 significant figures.) • loga ay = y
• loga 1 = 0
(a) log2 5 (b) log8 7 (c) log4 1
6 • loga a = 1
Without using a calculator, find the values of the following logarithms. • loga MN = loga M + loga N
[Nos. 2–7] • log a M = loga M - loga N
N

2. log3 27 ➥ Example 6 3. log2 32 4. log12 1 • loga M k = k loga M


log b M
• loga M =
5. log8 1 6. 1
log2 16 7.
3
log2 2 log b a
8 • If loga M = loga N,
then M = N.
Find the value of x in each of the following. [Nos. 8–11]
8. log2 x = 6 ➥ Example 7 9. log 3 x = -2
4

10. logx 16 = 2 11. logx 7 = -1

Without using a calculator, find the values of the following. [Nos. 12–20]

12. log8 2 + log8 4 ➥ Example 8(a) 13. log3 18 + log3 1 14. log2 6 - log2 3
2

log3 216
15. 3 log12 2 + log12 18 16. log8 2 - 2 log8 4 17.
log3 6
➥ Example 8(b)
log 6 4
18. 19. log16 64 ➥ Example 8(c) 20. log 7 49
log 6 256

Simplify the following expressions, where x 2 0 and x ! 1. [Nos. 21–24]


log5 x
21. 22. log2 4x + log2 x
log5 x
23. (log x)(logx 10) 24. (log9 x)(logx 3)

25. If log2 12 = a, express the following logarithms in terms of a.


(a) log2 24 (b) log2 3 (c) log2 144
Exam-type
26. If log3 2 = a and log3 5 = b, express the following logarithms in terms of a and b.
(a) log3 20 (b) log3 10 (c) log2 5

Level 2
Without using a calculator, find the values of the following logarithms. [Nos. 27–29]

29. log 7 7
4
27. log5 5-1 28. log3 27
7
Logarithmic Functions | 8.19

Without using a calculator, find the values of the following. [Nos. 30–37] ✂

Non-foundation
30. log 9 27 + log 9 3 31. 8 log12 2 + 2 log12 3

32. log4 2 + log4 40 - log4 5 33. 4 log3 2 - log3 6 - log3 8

7 2
34. (log8 81)(log9 64) 35. (log 5 ) (log5 1 000)
3
log 8
36. (log6 10)(log 6 ) 37.
log100 64

Simplify the following expressions, where x 2 0 and x ! 1. [Nos. 38–41]


2 + log3 x log2 (4x)
38. 39. 9 7
log3 (9x) log2 (16x ) - log2 x
log2 x
40. 2 log9 x - log3 x ➥ Example 9(a) 41.
log8 x

Simplify the following expressions, where x 2 0, y 2 0 and x ! 1, y ! 1. [Nos. 42–44]


2 2
2 3 log x y log y x - (log y 2) (log2 x)
42. (logy x )(logx y ) ➥ Example 9(b) 43. 44.
8 log x y log y x

Let x 2 0 and y 2 0. Express y in terms of x. [Nos. 45–48]

45. log5 x = 2 log5 y ➥ Example 10 46. 1 log y = 1 + log x


8 8
2

47. log2 y = 2 log4 x 48. log3 x - 2 = log9 y


Exam-type
49. If log5 2 = a and log5 3 = b, express the following logarithms in terms of a and b.

(a) log25 6 (b) log 1 18


5

Exam-type
50. If log x = a and log y = b, express the following in terms of a and b,
where x 2 0, y 2 0 and x ! 1, y ! 1.
(a) log x xy
8
(b) log y x3
Exam-type p q
51. It is given that 3 = 2 and 7 = 2 . Express log7 3 in terms of p and q.
Exam-type p q
52. It is given that 6 = 3 and 8 = 3 . Express log6 288 in terms of p and q.

Level 3

53. It is given that log8 y = - 1 log4 x + 1 . Express y in terms of x.


3 6

54. If log12 2 = r, express log12 9 in terms of r.

55. Show that 1 + 1 = 1, where x 2 0, y 2 0 and x ! 1, y ! 1.


log x (xy) log y (xy)

E_SrSecMath_4B08.indd 19 22/5/2023 下午2:45


8. 20 | Chapter 8


8.3 Logarithmic Functions and Their Graphs
Non-foundation

A Logarithmic Functions
Consider y = log2 x.
When x = 0.5, y = log2 0.5 = -1.
When x = 1, y = log2 1 = 0.
When x = 2, y = log2 2 = 1.
When x = 3, y = log2 3 = 1.58, cor. to 3 sig. fig.
h

For each value of x (x 2 0), there is one (and only one) corresponding x y
value of y. Hence, y = log2 x is a function of x.
0.5 –1
In general: 1 0
2 1
3 1.58...
Let a be a constant, a 2 0 and a ! 1. .. ..
. .
The function y = loga x or f(x) = loga x is called a
logarithmic function with base a. Function y = log2 x
(x is an independent variable
and y is a dependent variable)
e.g. y = log3 x, y = log 1 x and y = log1.5 x are logarithmic functions.
2

Note: When x G 0, loga x is undefined. Hence, the domains of logarithmic


functions are all positive real numbers.

B Graphs of Logarithmic Functions Concept Tool


(Graphs of
I. Graphs of y = loga x, where a 2 1 Logarithmic Functions)

Class Activity 8.2


Objective: To explore the features of the graphs of logarithmic functions.

1. According to the graph of y = log2 x on the right, y

answer the following questions.


2
y = log2 x
(a) (i) The x-intercept of the graph is .
1
(ii) The graph ( cuts / does not cut ) the y-axis,
and it lies on the ( left / right ) of the y-axis.
x
(b) (i) The graph slopes ( upwards / downwards ) 0 1 2 3 4 5
from left to right.
–1
(ii) When the value of x increases, the value of
y ( increases / decreases ). –2

logarithmic function 對數函數


Logarithmic Functions | 8. 21

2. (a) Consider the logarithmic function y = log4 x. Complete the following table.

Non-foundation
(Give the answers correct to 1 decimal place if necessary.)
x 0.3 0.5 1 2 3 4 4.5
y

(b) Plot the graph of y = log4 x in the given figure on the previous page.

3. According to the graph of y = log4 x, answer the following questions.


(a) (i) The x-intercept of the graph is .
(ii) The graph ( cuts / does not cut ) the y-axis, and it lies on the ( left / right ) of the y-axis.
(b) (i) The graph slopes ( upwards / downwards ) from left to right.
(ii) When the value of x increases, the value of y ( increases / decreases ).

4. According to Questions 1 and 3, do the graphs of the functions


y = log2 x and y = log4 x have common features? Yes No

From Class Activity 8.2, we can observe some features of the graphs of logarithmic functions. In general,
y = loga x (a 2 1) and their graphs have the following features:
y
① The x-intercept of the graph is 1. y = log a x
② The graph does not cut the y-axis. ③
It lies on the right of the x-axis.
When 0 1 x 1 1, y 1 0. When
x 2 1, y 2 0. ①
x
③ The graph slopes upwards from 0 1
left to right.

In addition, we can substitute different values of x into y = loga x (a 2 1) to observe the change in the rate
of increase of y when x increases. The table below shows the case of y = log x.
+1 +1 +1

x 1 2 3 4
y 0 0.301g 0.477g 0.602g
The rate of increase
of y decreases
+0.30g +0.17g +0.12g

We can see that:

④ When the value of x increases, the


value of y increases and the rate of
increase of y decreases.

In fact, this is another feature of y = loga x (a 2 1).


8. 22 | Chapter 8

✂ II. Graphs of y = loga x, where 0 1 a 1 1


Non-foundation

The figure on the right shows the graphs of y = log 1 x and y


2
3
y = log 1 x.
4
2

From the figure, the graphs of y = log 1 x and y = log 1 x 1


2 4
have similar features to the graphs of y = log2 x and x
0 1 2 3 4 5 6 7 8 9
y = log4 x.
–1

–2 y = log 1 x
4

–3 y = log 1 x
2

In general, y = loga x (0 1 a 1 1) and their graphs have the following features:

y
① The x-intercept of the graph is 1.
② The graph does not cut the y-axis. ②
It lies on the right of the y-axis.
When 0 1 x 1 1, y 2 0. When
x 2 1, y 1 0.

③ The graph slopes downwards x
0
1
from left to right.

y = log a x

In addition, we can substitute different values of x into y = loga x (0 1 a 1 1)


to observe the change in the rate of decrease of y when x increases. The table
below shows the case of y = log0.5 x.
+1 +1 +1

x 1 2 3 4
y 0 -1 -1.584g -2
The rate of decrease
of y decreases
–1 –0.58g –0.41g

We can see that:

④ When the value of x increases, the


value of y decreases and so does
the rate of decrease of y.

In fact, this is another feature of y = loga x (0 1 a 1 1).

E_SrSecMath_4B08.indd 22 22/5/2023 下午2:45


Logarithmic Functions | 8. 23

III. Comparing the Graphs of y = loga x and y = log1 x ✂


a

Non-foundation
The figure below shows the graphs of y = log2 x and y = log 1 x.
2
y
y = log 2 x
3

x
0 1 2 3 4 5 6 7 8
–1

–2
y = log 1 x
2
–3

From the above figure, we observe that the graph of y = log 1 x is the
2
reflection image of the graph of y = log2 x with respect to the x-axis.
In fact:

For a 2 0 and a ! 1, the graphs of y = log 1 x and y = loga x are


a
the reflection images of each other with respect to the x-axis.

From the above discussions, we can summarize the features of the


logarithmic functions y = loga x and their graphs:

Range of a a21 0 1 a 11
y y
y = log a x

Graph of
y = loga x x x
0 1 0 1
y = log a x

1. The x-intercept of the graph is 1.


Common 2. The graph does not cut the y-axis. It lies on the right of the y-axis.
features 3. The graphs of y = log 1 x and y = loga x are the reflection images of each other
a
with respect to the x-axis.

1. When 0 1 x 1 1, y 1 0. 1. When 0 1 x 1 1, y 2 0.
When x 2 1, y 2 0. When x 2 1, y 1 0.
2. The graph slopes upwards from left 2. The graph slopes downwards from
Different
to right. left to right.
features
3. When the value of x increases, the 3. When the value of x increases, the
value of y increases and the rate of value of y decreases and so does the
increase of y decreases. rate of decrease of y.

E_SrSecMath_4B08.indd 23 22/5/2023 下午2:45


8. 24 | Chapter 8

✂ Quick Quiz 8.6


Non-foundation

Refer to the figure on the right. In each of the following,


put ‘✓’ in if it is correct, and ‘✗’ if it is wrong. y
1. The value of b is 1.
2. The range of values of a is a 2 1. y = loga x
3. The graph of y = loga x does not cut the y-axis.
x
O
b
4. The graph of y = log 1a x is the reflection image of the
graph of y = loga x with respect to the y-axis.

Compare the Bases from the Given Graphs


For two logarithmic functions with different bases, there are two cases
for their graphs.

Case 1: One graph slopes downwards from left to right, and the
other slopes upwards from left to right

The figure shows the graphs of y = logp x and y = logq x. y

From the shapes of the graphs, y = logq x


we obtain 0 1 p 1 1 and q 2 1, i.e. p 1 q .
x
O

y = logp x

Case 2: Both graphs slope downwards (or upwards) from left to


right
y
The figure shows the graphs of y = logm x and y = logn x. x=2
From the shapes of the graphs, we can only obtain 0 1 m 1 1
and 0 1 n 1 1.
We can compare the values of the functions at a certain value of x x
O
(e.g. x = 2).
log n 2 y = log n x
Then logm 2 1 logn 2 log m 2
log 2 log 2 y = log m x
1 ◀ Note that log 2 2 0, log m 1 0 and log n 1 0.
log m log n
1 1 1 ◀ Since log 2 2 0, the inequality sign remains unchanged.
log m log n
log n 1 log m ◀ >;;7;;;?
Multiply both sides by (log m)(log n).

Thus, n 1 m. ◀
20
We will further discuss this property on P. 8.37.

Example 11 will show more techniques on solving problems of this


kind.
Logarithmic Functions | 8. 25

[Solve a problem related to the graphs of logarithmic functions]



y L:x=k

Non-foundation
Example The figure shows the graphs of y = loga x and y = log x,
Level 2
11
where a is a positive constant. A vertical line L: x = k
C x
cuts the graph of y = loga x, the graph of y = log x and the O
x-axis at A, B and C respectively. y = log x B y = loga x
Explain
(a) Determine whether each of the following is true. A
Explain your answers.
(i) k 1 a (ii) a 2 10
AC
(b) Express BC in terms of a.

Solution (a) (i) From the figure, 0 1 k 1 1 ◀ The x-intercepts of both graphs are 1.
and a 2 1. ◀ The graph of y = loga x slopes
` k 1 a is true. upwards from left to right.

(ii) From the figure, when x = k,


loga k 1 log k ◀ y L:x=k
log k y = log x
1 log k C
log a x
O
1 2 1 ◀ Since log k 1 0, the inequality log k y = loga x
B
log a sign should be reversed .
1 2 log a ◀ Multiply both sides by >7?
log a. loga k A

log 10 2 log a 20

10 2 a
` a 2 10 is not true.

(b) AC = -loga k, BC = -log k.


AC - log a k
BC = - log k
log k
log a
=
log k
Exam Video
= 1
log a (for DSE Level 5**)

Instant Drill 11
The figure shows the graphs of y = log0.1 x y L:x=k
and y = loga x, where a is a positive constant.
A vertical line L: x = k cuts the graph of
y = log0.1 x, the graph of y = loga x and the A
x-axis at A, B and C respectively.
y = log0.1 x
(a) Determine whether each of the following B
is true. Explain your answers. x
O C y = loga x
(i) ak 2 1 (ii) a 1 0.1
➥ Ex 8C 11, 12
(b) Express BC
AC
in terms of a.
★ Public Exam Question 8

E_SrSecMath_4B08.indd 25 22/5/2023 下午2:45


8. 26 | Chapter 8

✂ C Relationship between the Graphs of


Exponential and Logarithmic Functions
Non-foundation

The following figure shows the graphs of y = 2x and y = log2 x.


y
y = 2x y=x
(2,4)
4

3
y = log 2 x
( 1, 2)
2
( 4 ,2 )

1
( 2, 1)

x
–1 0 1 2 3 4 5
–1

When (2 , 4) and (1 , 2) are reflected with respect to the line y = x, the


images are (4 , 2) and (2 , 1) respectively, i.e. the x-coordinate and the
y-coordinate of each point are swapped.

If we swap the x-coordinate and the y-coordinate of every point on the ◀ y = 2x


x
graph of y = 2 , the graph of y = log2 x is obtained. Hence, the graph of x -1 0 1 2
x
y = log2 x is the reflection image of the graph of y = 2 with respect to the
y 0.5 1 2 4
line y = x.
y = log2 x
x
The graphs of y = 0.5 and y = log0.5 x also have this relationship. x 0.5 1 2 4

y y -1 0 1 2
y = 0.5 x y=x
( –2,4) 4

2
(–1, 2) ◀ y = 0.5
x

1 x -2 -1 0 1

x y 4 2 1 0.5
–2 –1 0 1 2 3 4
y = log0.5 x
–1 ( 2, –1)
x 4 2 1 0.5
y = log 0.5 x
–2
( 4 ,– 2 ) y -2 -1 0 1

In general:

For a 2 0 and a ! 1, the graphs of y = ax and y = loga x are


the reflection images of each other with respect to the line y = x.

E_SrSecMath_4B08.indd 26 22/5/2023 下午2:45


Logarithmic Functions | 8. 27

For a 2 0 and a ! 1, the following table summarizes the features of the ✂


x
exponential functions y = a , logarithmic functions y = loga x and their

Non-foundation
graphs.

y = ax y = loga x
Range of a a21 01a11 a21 01a11

y = a 41 k
x
Example y = 7x y = log3 x y = log 1 x
5

y y y y
y = ax y = ax
y = loga x
Graph x
1 0 1
1 x
x x 0 1
0 0 y = loga x

Intersection
No points of intersection (1 , 0)
with the x-axis
Intersection
(0 , 1) No points of intersection
with the y-axis
Direction of
slope from left Upwards Downwards Upwards Downwards
to right
Change in
y when x Increase Decrease Increase Decrease
increases
*Rate of
increase/
Rate of increase Rate of decrease Rate of increase Rate of decrease
decrease
increases decreases decreases decreases
of y when x
increases
When reflected
The graph of y = ` 1a j is obtained when
x
with respect The graph of y = log 1 x is obtained when
a
to the x-axis/ reflected with respect to the y-axis. reflected with respect to the x-axis.
y-axis
When reflected
x
with respect to The graph of y = loga x is obtained. The graph of y = a is obtained.
the line y = x

*For the explanation of the rates of increase and decrease, students may
refer to P. 7.15 and P. 7.16 in Chapter 7, and P. 8.21 and P. 8.22 in this
chapter.

E_SrSecMath_4B08.indd 27 22/5/2023 下午2:45


8. 28 | Chapter 8

✂ Class Practice 8.3


Non-foundation

1. Write down the corresponding function for each of the following graphs.

y = 1.3x, y = log0.3 x, y = log1.3 x, y = 0.3x

(a) y (b) y (c) y (d) y

x 1 1 x
0 1 x x 0 1
0 0

2. The figure shows the graphs of y = logm x, y = logn x and y = log6 x, y

where m and n are positive constants. y = log6 x

(a) Determine whether the value of m is greater than or less than 6.


y = logm x
(b) If the graph of y = logn x is the reflection image of the graph
x
of y = log6 x with respect to the x-axis, find the value of n. O
y = logn x

Exercise 8C
[In this exercise, the softcopies of the figures for Questions 6 and 7 can be
downloaded from the publisher’s website.]

Level 1

1. In each of the following, the graph of the logarithmic function is reflected with respect to
the x-axis. Write down the corresponding function of each graph obtained.
(a) y = log7 x (b) y = log 1 x (c) y = log0.2 x (d) y = log 3 x
4 2

In each of the following, the graph of the function is reflected with respect to the line y = x.
Write down the corresponding function of each graph obtained. [Nos. 2–3]

(c) y = ` 1 j
x x x
2. (a) y = 7 (b) y = 0.2 (d) y = ( 3 ) x
6

3. (a) y = log2 x (b) y = log 1 x (c) y = log0.8 x (d) y = log 5x


3

4. The two curves in the figure represent the graphs of the logarithmic y
functions y = log3 x and y = log0.5 x.

(a) Write down the corresponding logarithmic functions for C1 and C2. C1

(b) C1 and C2 cut the x-axis at a point. Find the coordinates of that x
O
point.

C2
Logarithmic Functions | 8. 29

y
5. The two curves in the figure represent the graphs of the functions C3

y = log x and y = 4x. Write down the corresponding functions for C3

Non-foundation
and C4.

C4

x
O

According to the graph given in each of the following figures, sketch the
graph of the required function in the same figure. [Nos. 6–7]
6. (a) y = log 1 x (b) y = log5 x
3
y y

2 2
y = log 3 x
1 1

x x
0 1 2 3 4 0 1 2 3 4

–1 –1 y = log 1 x
5

–2 –2

x
7. (a) y = 6 (b) y = log0.4 x
y y
y=x y=x
3 3

2 2

1 y = log6 x 1
y = 0.4x
x x
–2 –1 0 1 2 3 4 0
–2 –1 1 2 3 4
–1 –1

–2 –2

Level 2
y
8. The figure shows the graphs of y = loga x and y = logb x, where a and b
are positive constants. It is given that the graph of y = loga x passes y = loga x
(25 ,2)
through (25 , 2).
(a) Find the value of a.
x
O
(b) If the graph of y = logb x is the reflection image of the graph of
y = logb x
y = loga x with respect to the x-axis, find the value of b.

E_SrSecMath_4B08.indd 29 22/5/2023 下午2:45


8. 30 | Chapter 8

Exam-type y
✂ 9. The figure shows the graphs of y = loga x, y = logb x and y = log 1 x ,
3 y = log b x
Non-foundation

where a and b are positive constants. The graph of y = loga x is the


reflection image of the graph of y = log 1 x with respect to the x-axis. y = log a x
3

(a) Find the value of a. x


O

(b) Arrange a, b and 1 in descending order.


3 y = log 1 x
3

Exam-type
10. The figure shows the graphs of y = log8 x, y = logm x and y = logn x, y

where m and n are positive constants. The three graphs cut the x-axis
y = log 8 x
at the point Q.
(a) Find the coordinates of Q.
x
O Q
(b) It is given that the graph of y = log8 x is the reflection image of y = log m x
the graph of y = logm x with respect to the x-axis.
y = log n x
(i) Find the value of m.
(ii) Find the range of values of n.
Exam-type
11. The figure shows the graphs of y = loga x and y = logb x, where a and b y
L:x=k
are positive constants. A vertical line L: x = k cuts the graph of y = log a x
y = loga x, the graph of y = logb x and the x-axis at A, B and C A
y = log b x
respectively. B
C
Explain (a) Determine whether each of the following is true. x
O
Explain your answers.
2
(i) a+b22 (ii) a 2 ab

(b) Show that AC = loga b. ➥ Example 11


BC
Exam-type
12. The figure shows the graphs of y = loga x and y = logb x, y
L:x=k
where a and b are positive constants. A vertical line L: x = k y = logb x
C x
cuts the graph of y = loga x, the graph of y = logb x O
B
and the x-axis at A, B and C respectively. If AB = m, where y = loga x
BC
m is a positive constant, show that b = am + 1. A

13. In the figure, C is the reflection image of the graph of y = log4 x with y
C
respect to the line y = x. R
(a) Write down the corresponding function for C. y=x

(b) It is given that the x-coordinate of the point R is 2, and the point S
is the reflection image of R with respect to the line y = x. Find the y = log4 x
coordinates of S. x
O

E_SrSecMath_4B08.indd 30 22/5/2023 下午2:45


Logarithmic Functions | 8. 31

Level 3 ✂

Non-foundation
x
14. The figure shows the graph of y = 0.2 . Using the given figure, find the y
4
value of log5 3, correct to 1 decimal place.

1
y = 0.2 x

x
–1 0 1 2

8.4 Exponential Equations and Logarithmic Equations


A Exponential Equations
Equations with unknown indices are called exponential equations.
x+1
x
e.g. 2 = 5, 3 = 3 and 3x - 2 = 6x are exponential equations. ◀ Special types of exponential
equations will be discussed in
For a simple exponential equation, we can change both sides of the equation Book 5A Chapter 1.
to powers of the same base and use the following property to find the solution:

Let a 2 0 and a ! 1. If ax = ay, then x = y.

[Solve exponential equations]

Example Solve the following exponential equations.


Level 1
12 x+1
(a) 5 = 53 - 2x (b) 3
x-1
= 27

Solution (a) 5x + 1 = 53 - 2x
` x + 1 = 3 - 2x
3x = 2
x = 23

(b) 3x - 1 = 27
Key
3x - 1 = 3 3
Rewrite the equation so that the
3
3x - 1 = 3 2 expressions on both sides have
the same base 3.
` x-1= 3
2

x= 5 Exam Video
2
(for DSE Level 4+)

E_SrSecMath_4B08.indd 31 22/5/2023 下午2:45


8. 32 | Chapter 8

✂ Instant Drill 12
Non-foundation

Solve the following exponential equations.


x+3 3
(a) 6 = 61 - x (b) 5
2x
= 25
➥ Ex 8D 1–8
x-2 x+1
(c) 9 =3 (d) 2 = 4x - 1
★ Public Exam Question 3

[Solve exponential equations]


x x+1
Example Solve 5 + 5 = 30.
Level 2
13

Solution 5x + 5x + 1 = 30
x x
5 + 5 :5 = 30 ◀ Rewrite 5x + 1 as 5x:5.
x
5 (1 + 5) = 30 ◀ Extract the common factor 5x.
x
5 :6 = 30
x
5 =5
x= 1

Instant Drill 13
Solve the following exponential equations.
x x+1 x+2
(a) 4 + 4 = 80 (b) 3 - 3x = 24
➥ Ex 8D 30–32

When it is difficult to change both sides of an exponential equation to


powers of the same base, we can use logarithms to find the solution.

[Solve an exponential equation using logarithms]


x
Example Solve 6 = 12 and give the answer correct to 3 significant
Level 1
14
figures.

Solution 6x = 12
x Key
log 6 = log 12
Take common logarithms on both
x log 6 = log 12 ◀ log M k = k log M sides of the equation.
log 12 log 12
x= ◀ ! log 2
log 6 log 6
= 1.39 , cor. to 3 sig. fig.

Instant Drill 14
Solve the following exponential equations and give the answers correct
to 3 significant figures.
(b) ` 1 j = 8
x x
(a) 7 = 14
3
➥ Ex 8D 9–14

E_SrSecMath_4B08.indd 32 22/5/2023 下午2:45


Logarithmic Functions | 8. 33

[Solve exponential equations using logarithms]


Non-foundation
Example Solve the following exponential equations and give the
Level 2
15
answers correct to 3 significant figures.
x+1 1 - 3x
x
(a) 3 = 4 (b) 7(2 ) = 32x

Solution (a) 3x = 4x + 1
log 3x = log 4x + 1
x log 3 = (x + 1) log 4
x log 3 = x log 4 + log 4
x log 3 - x log 4 = log 4
x(log 3 - log 4) = log 4 Calculator
Keying sequence:
log 4
x= ◀ log 4 ) ' ( log 3 )
log 3 - log 4
– log 4 ) ) EXE
= - 4.82 , cor. to 3 sig. fig.

(b) 7(21 - 3x) = 32x


log [7(21 - 3x)] = log 32x
log 7 + log 21 - 3x = log 32x ◀ log MN = log M + log N
log 7 + (1 - 3x) log 2 = 2x log 3
log 7 + log 2 - 3x log 2 = 2x log 3
log 7 + log 2 = 3x log 2 + 2x log 3 Calculator
Keying sequence:
log 7 + log 2 = x(3 log 2 + 2 log 3)
( log 7 ) + log 2 )
log 7 + log 2 ◀ ) ' ( 3 # log 2 )
x=
3 log 2 + 2 log 3 + 2 # log 3 ) ) EXE

= 0.617, cor. to 3 sig. fig.

Instant Drill 15
Solve the following exponential equations and give the answers correct
to 3 significant figures.
1-x x-1
x
(a) 2 = 3 (b) 9 = 72x x
(c) 4(5 ) = 3
x-1

➥ Ex 8D 33–38

B Logarithmic Equations
Equations with unknowns in logarithms are called logarithmic equations.
e.g. log x = 1, log0.5 (x - 1) = 0 and log3 x + log4 x = 7 are logarithmic
equations. ◀ Special types of logarithmic
equations will be discussed in
Book 5A Chapter 1.

E_SrSecMath_4B08.indd 33 22/5/2023 下午2:45


8. 34 | Chapter 8

✂ [Solve logarithmic equations]


Non-foundation

Example Solve the following logarithmic equations.


Level 1
16
(a) 2 log4 x - 3 = 0
(b) log5 (x + 1) = 2 log5 2

Solution (a) 2 log4 x - 3 = 0


Key
log4 x = 3 ◀ ①
2 By the definition of logarithms:
3
` x=4 2 ◀
① Rewrite the equation in the form

loga x = p .
3
= (22) 2 ② Use the definition to obtain x = a .
p

=8

(b) log5 (x + 1) = 2 log5 2


2 Key
log5 (x + 1) = log5 2 ◀ ①
By the property of logarithms:
` x + 1 = 22 ◀ ②
① Rewrite the equation in the form
x=3 loga x = loga q .
② Use the following property:
If loga x = loga q , then x = q .

Instant Drill 16
Solve the following logarithmic equations.
(a) 6 log x - 12 = 0 (b) 3 log8 x + 2 = 0
(c) log (x + 1) = 2 log 2 (d) log6 (x - 1) = 1 log6 4
2
➥ Ex 8D 15– 23

Try to Organize

A student uses the two techniques illustrated in Example 16 to solve


equations. However, the student cannot obtain the correct answers. Do
you know why?

By the definition of logarithms By the property of logarithms


2 log4 x - 3 = 0 log (1 + x) = log3 9
log4 x2 - 3 = 0 1+x=9
log4 x2 = 3 x=8

x2 = 43
x2 = 64
x = 8 or - 8 ✗
Logarithmic Functions | 8. 35

[Solve logarithmic equations] ✂

Non-foundation
Example Solve the following logarithmic equations.
Level 2
17
(a) log2 x + log4 x = 6 (b) log (3x - 1) = 1 + log (x + 2)

Solution (a) log2 x + log4 x = 6


log2 x log b M
log2 x + =6 ◀ loga M =
log2 4 log b a
log2 x
log2 x + 2 = 6
log2 2
log2 x
log2 x + =6 ◀ loga ay = y
2
3 log x = 6
2 2

log2 x = 4
4
` x=2
= 16

(b) log (3x - 1) = 1 + log (x + 2)


log (3x - 1) = log 10 + log (x + 2) ◀ log 10 = 1
log (3x - 1) = log [10(x + 2)] ◀ log M + log N = log MN
` 3x - 1 = 10x + 20
-7x = 21
x = -3 (rejected) ◀ When x = -3,
` The equation has no real solutions. log (3x - 1) = log (-10) and
log (x + 2) = log (-1), which
are undefined.
Note: We must check the answer after solving a logarithmic equation to
see whether every logarithm in the equation is defined.

Instant Drill 17
Solve the following logarithmic equations.
(a) log3 x + log27 x + 4 = 0 (b) log (x - 10) = log (x - 1) + 1
➥ Ex 8D 39–46

Class Practice 8.4


Without using logarithms, solve the following exponential equations. [Nos. 1–3]
2x + 3 3-x 2x 1-x x+1 x
1. 3 =3 2. 5 = 25 3. 7 - 7 = 42

Solve the following exponential equations and give the answers correct to 3 significant figures.
[Nos. 4–6]
x x-1 x 1+x
4. 2 = 10 5. 6 =7 6. 3 = 10

Solve the following logarithmic equations. [Nos. 7–9]


7. log2 (3x) = 0 8. log (x + 1) = 2 log 3 9. log (5x) = log (2x + 5) + 1
8. 36 | Chapter 8

✂ Exercise 8D
Non-foundation

Level 1
Without using logarithms, solve the following exponential equations. [Nos. 1–8]
1-x x+7
1. 3 = 3x ➥ Example 12 2. 4 = 49 - x
x
2x - 1 5+x x-2
3. 7 =7 4. 3 = 32
Exam-type 6-x Exam-type x+1 3
5. 3 = 81 6. 10 = 10

Exam-type Exam-type
7. 2
x+3
= 32 8. 25
x-2
= 1
125

Solve the following exponential equations and give the answers correct to
3 significant figures. [Nos. 9–14]
x x
9. 2 = 6 ➥ Example 14 10. 400 = 3

x x-1
11. 1.5 = 15 12. 3 =4
x+1 2x
13. 8 = 70 14. 7 = 12

Solve the following logarithmic equations. [Nos. 15–23]

15. log (x + 5) = 1 ➥ Example 16 16. 2 log9 x - 1 = 0 17. log2 (2x + 3) = 0

18. log (x - 1) = 4 log 3 19. log4 (6 - x) = 2 log4 5 20. log5 x - 1 log5 8 = 0


3

21. log (x - 1) - log 3 = log 5 22. log2 x - log2 3 = 2 23. log (x + 1) - log (1 - x) = 1

Level 2
Without using logarithms, solve the following exponential equations. [Nos. 24–32]

= a1k
x-3 2-x
= 25-x
x+2
= 94 - x
2x
24. 5 25. 3 26. 2
8
2x x
4 x 4-x x +1
27. 1 - x
=4 28. 4 :8 =1 29. 3:9 = 27 2
4
x+1 x-2 x-1
30. 2 + 2x = 24 ➥ Example 13 31. 4 - 4x = -15 32. 6 - 6x + 30 = 0

Solve the following exponential equations and give the answers correct to
3 significant figures. [Nos. 33–38]
1-x x+1
= 52 - x
2+x x
33. 6 = 11x ➥ Example 15 34. 7 = 3 35. 4

x+1 2x - 1
) = 22x
x+3
= 20(1.56x - 1)
x
36. 6(8 ) = 9 37. 3(7 38. 5
Logarithmic Functions | 8. 37

Solve the following logarithmic equations. [Nos. 39–46] ✂

Non-foundation
39. log100 x - log x = 1 ➥ Example 17 40. log3 x + log9 x = 3

41. log8 x - log2 x = -4 42. log25 x - log5 x - 1 = 0

43. log (x + 3) - 1 = log x 44. log (2x) + 1 = log (5x - 10)

45. log4 (x - 15) = log4 x + 2 46. log2 (x + 5) - 3 = log2 (4 - x)

Level 3
47. Solve the following simultaneous equations.
2
)
x + logr y = -6
x - 5 logr y = 8
(Give the answers in terms of r if necessary.)
3
48. Solve the equation 3 log8 (4x) + 2 = log2 x .

8.5 Applications of Logarithms


A Basic Applications
Which one is greater? We
cannot evaluate them with We can use logarithms
a calculator f to compare their values.

456
123
456123

We know that the graph of y = log x slopes upwards from left to right, y
i.e. when the value of x increases, the value of y increases, and vice versa.

In other words, y = log x

when M 2 N 2 0, log M 2 log N; x


0 1
when log M 2 log N, M 2 N.

The above property is useful in solving some application problems.

E_SrSecMath_4B08.indd 37 22/5/2023 下午2:45


8. 38 | Chapter 8

✂ [Use logarithms to compare values]

(a) Find the values of log 1 111340, log 2 222300 and log 3 333290.
Non-foundation

Example
Level 1
18
(Give the answers correct to 3 significant figures.)
340
(b) Among 1 111 , 2 222300 and 3 333290, which one is
the greatest?

Solution (a) log 1 111


340
= 340 log 1 111 = 1 040 , cor. to 3 sig. fig. ◀
k
log M = k log M
300
log 2 222 = 300 log 2 222 = 1 000 , cor. to 3 sig. fig.
290
log 3 333 = 290 log 3 333 = 1 020 , cor. to 3 sig. fig.
340 290 300
(b) a log 1 111 2 log 3 333 2 log 2 222 ◀ From (a).
340 290 300
` 1 111 2 3 333 2 2 222 ◀ If log M 2 log N, then M 2 N.
340
` 1 111 is the greatest.
Exam Video
(for DSE Level 4+)
Instant Drill 18
1. Among 123231, 231213 and 312132, which one is the least?
➥ Ex 8E 1–3
2. Among 5013 000, 4 001495 and 3 001500, which one is the greatest?
★ Public Exam Question 11

[Use logarithms to solve an application problem]


Example The original value of a car is $400 000. Its value decreases
Level 2
19
by 10% every year. At least how many years later will the
value of the car be less than $100 000?

Solution Suppose the value of the car will be less than $100 000
Key
n years later.
n If the value P decreases by R%
a 400 000(1 - 10%) 1 100 000
n in each period, then the new
` 0.9 1 0.25 value after k periods will be
n
log 0.9 1 log 0.25 P(1 - R%)k.
n log 0.9 1 log 0.25
log 0.25
n2 ◀ The sign is reversed since
log 0.9
log 0.9 1 0.
n 2 13.157g
` At least 14 years later, the value of the car will be less
than $100 000.

Instant Drill 19
1. The price of a phone is $5 000 now. Its price decreases by 2%
every month. At least how many months later will the price of the
phone be less than $3 000?
2. The value of an antique is $4 000 now. Its value increases by 9%
every year. At least how many years later will the value of the
antique be more than $8 000?
➥ Ex 8E 19–21

E_SrSecMath_4B08.indd 38 22/5/2023 下午2:45


Logarithmic Functions | 8. 39

B Logarithmic Transformation ✂

Non-foundation
Consider the graph on the right. We learnt that the equation of the straight y
line can be found with its slope and y-intercept:
y = mx + c slope = m
c
We say that x and y have a linear relation.
x
O
In some cases, the relation between two variables x and y is non-linear. For
x 5
example, consider the equations y = 3:4 and y = 2x (where x 2 0):
y y

y = 3.4x y = 2x5

x x
O O
y = 3:4x y = 2x5

Their graphs are both curves. However, we can take logarithms on both
sides of the equations and transform the non-linear relations to linear
relations. This method is called logarithmic transformation. The table
below shows two of the cases.

Case y = kax (where k 2 1 and a 2 1) y = bxn (where b 2 1 and n 2 1)


y y

y = ka x y = bx n
Original graph
k
x x
O O

y = kax y = bx
n
x n
log y = log (ka ) log y = log (bx )
Transformation log y = log k + log ax log y = log b + log xn
log y = log k + x log a log y = log b + n log x
log y log y
log y = log k + x log a log y = log b + n log x

slope = log a slope = n


Graph after
transformation log k log b

x log x
O O

Linear relation between x and log y Linear relation between log x and log y

Slope and intercept Slope = log a, Slope = n,


on the vertical axis intercept on the vertical axis = log k intercept on the vertical axis = log b

logarithmic transformation 對數變換

E_SrSecMath_4B08.indd 39 22/5/2023 下午2:45


8.40 | Chapter 8

✂ Note: We can take logarithms with an arbitrary base on both sides of


x
the equation. For example, y = ka can be transformed to
Non-foundation

logm y = logm k + x logm a, where m 2 0 and m ! 1.

[Find the unknowns in a relation of the form y = kax]

Example The graph in the figure shows the linear relation between log y
Level 1
20
x and log y. The intercepts on the vertical axis and on the 2
horizontal axis of the graph are 2 and 1 respectively. It is
x
given that y = ab , where a and b are constants.
(a) Express log y in terms of a, b and x. 0
x
1
(b) Hence, find the values of a and b.

Solution (a) y = ab
x
x
log y = log (ab ) ◀ Take common logarithm on both sides.
x
log y = log a + log b
log y = log a + x log b

(b) Key
① Horizontal axis: x, vertical axis: log y
② log y = log a + x log b
Intercept on the Slope
vertical axis

Intercept on the vertical axis = 2


log a = 2
2
a = 10
= 100
Slope = 01 - 02 ◀ The graph passes through (0 , 2) and (1 , 0).
-
Substitute log a = 2, x = 1 and
log b = -2 log y = 0 into log y = log a + x log b.
-2
b = 10 Exam Video 0 = 2 + log b
= 1 (for DSE Level 4+) -2 = log b
100 b = 10-2
= 1
100

Instant Drill 20
The graph in the figure shows the linear relation log y
(4 ,2 )
between x and log y. The graph passes through (4 , 2)
and its intercept on the vertical axis is -1. It is given
that y = abx, where a and b are constants.
x
(a) Express log y in terms of a, b and x. 0

(b) Hence, find the values of a and b. –1

(Give the answers correct to 3 significant figures ➥ Ex 8E 6, 7


if necessary.)
★ Public Exam Question 10

E_SrSecMath_4B08.indd 40 22/5/2023 下午2:45


Logarithmic Functions | 8.41

n
[Find the unknowns in a relation of the form y = bx ]

log3 y

Non-foundation
Example The graph in the figure shows the linear relation between
Level 1
21 (– 4 ,1 )
log3 x and log3 y. The graph passes through (-4 , 1) and the
intercept on the vertical axis of the graph is -3. It is given log3 x
0
b
that y = ax , where a and b are constants. Find the values of
a and b.
–3

Solution Key
① Horizontal axis: log3 x, vertical axis: log3 y

② Express log3 y in terms of a, b and log3 x:


log3 y = log3 a + b log3 x
Intercept on the Slope
vertical axis

y = ax b
log3 y = log3 (axb)
b
log3 y = log3 a + log3 x
log3 y = log3 a + b log3 x
Intercept on the vertical axis = -3
log3 a = -3
-3
a=3
= 1
27

Slope = - 3 - 1 ◀ The graph passes through (-4 , 1) and (0 , -3). Exam Video
0 - (- 4)
b = -1 (for DSE Level 4+)

Instant Drill 21
The graph in the figure shows the linear relation log2 y

between log 2 x and log 2 y. The intercepts on the


horizontal axis and the vertical axis of the graph are 3

-2 and 3 respectively. It is given that y = axb, where a


and b are constants. Find the values of a and b.
log2 x
–2 0
➥ Ex 8E 8, 9

★ Public Exam Questions 2, 9

E_SrSecMath_4B08.indd 41 22/5/2023 下午2:45


8.42 | Chapter 8

✂ [Find a non-linear relation]


Non-foundation

Example It is given that log2 y is a linear function of log2 x. The graph


Level 2
22
of the linear function passes through (0 , 3) and (2 , 11).
Express y in terms of x.

Vertical axis Horizontal axis

Solution log2 y - 3 = 11 - 3 (log2 x - 0) ◀ Two-point form of


2-0
straight lines:
log2 y - 3 = 4 log2 x log2 y
log2 y = 4 log2 x + 3
( 2 ,11 )
4 3
log2 y = log2 x + log2 2
4 Exam Video
log2 y = log2 (8x )
4 ( 0 ,3 ) (for DSE Level 5**)
y = 8x log2 x
O

Instant Drill 22
1. It is given that log2 y is a linear function of
log2 x. The slope and the intercept on the
slope =
horizontal axis of its graph are 31 and 6
respectively. Express y in terms of x. 0

[Hint: Fill in the blanks in the figure on the right first.]


2. It is given that log5 y is a linear function of x. The graph of the linear ➥ Ex 8E 24, 25
function passes through (1 , 1) and (3 , 9). Express y in terms of x.
★ Public Exam Question 7

More to Learn

Non-linear Relations Involving Surds


For non-linear relations involving surds, we can find the relation between two variables by a method
similar to the one in Example 22.
2
e.g. The graph in the figure shows the linear relation between x and y , where y H 0.
y - 8 = 0 - 8 (x - 0)
2 y
4-0
2 8
y = -2x + 8
2 2
y = (-2x + 8)
y = 4x4 - 32x2 + 64
x2
0 4
In each of the following, express y in terms of x.
(a) y (b) y

4
3

x x3
–2 0 0 1
Logarithmic Functions | 8.43

C Intensity Level of Sound ✂

Non-foundation
In Physics, the unit for the intensity of sound is W/m2. Since the intensity
-12
of sound may vary greatly in order of magnitude (from 10 W/m2 to Interesting Maths
2
100 W/m ), we often use another method to express the loudness of sound: Loudness of different sounds:
the intensity level.
Quiet whisper 30 dB
For a sound of intensity I, its intensity level D decibels (dB) is defined as Quiet street corner 50 dB
follows: Normal conversation 60 dB
Loud singing 75 dB
D = 10 log II
0 Busy traffic 90 dB
Disco 100 dB
where I0 represents the intensity of the sound that we can just hear (the Thunder 120 dB
-12
so-called threshold of hearing) and I0 = 10 W/m2. Aeroplane 130 dB
-5
e.g. If the intensity of a certain sound is 10 W/m2, then More information about noise
-5
the intensity level of the sound = 10 log 10-12 dB ◀ I0 = 10-12 W/m2
can be found in the website of
10 the Environmental Protection
Department.
= 10 log 107 dB
= 10:7 dB
= 70 dB
I
The intensity level of the sound that we can just hear is 10 log I0 dB, i.e. 0 dB.
0

[Solve a problem related to the intensity level of sound]

Example The intensity level of a sound produced by an electric


Level 1
23
fan is 48 dB. Find the intensity of the sound, correct to
3 significant figures.
2
Solution Let I W/m be the intensity of the sound.
48 = 10 log I
-12
10
4.8 = log I
-12
10
` I = 104.8
-12
10
-12 4.8
I = 10 # 10
-7.2
= 10
-8
= 6.31 # 10 , cor. to 3 sig. fig.
-8 2
` The intensity of the sound is 6.31 × 10 W/m .

Instant Drill 23
The intensity level of a sound produced by an aeroplane when taking off is
130 dB. Find the intensity of the sound.
➥ Ex 8E 13, 14

intensity of sound 聲音強度 intensity level 音量

E_SrSecMath_4B08.indd 43 22/5/2023 下午2:45


8.44 | Chapter 8

✂ D Richter Scale
Non-foundation

The magnitude of an earthquake is commonly measured by the Richter


scale. This is a quantitative measure invented by an American seismologist Interesting Maths
Charles Francis Richter (1900-1985) in 1935. If M is the magnitude of More information about
an earthquake on the Richter scale, then the relation between M and the earthquakes can be found in
energy E (in joules or J) released in the earthquake is as follows: the website of the Hong Kong
Observatory.

M = 2 log E + K, where K is a constant.


3

Note: In this chapter, the magnitudes of earthquakes involved in the questions


are measured by the Richter scale.

[Solve a problem related to the Richter scale]

Example The magnitudes of the Kumamoto Earthquake (2016) and


Level 2
24
the Great East Japan Earthquake (2011) were 7.0 and 9.0
respectively. How many times the energy released in the
Kumamoto Earthquake was the energy released in the Great
East Japan Earthquake?

Solution Let E1 J and E2 J be the energies released in the Kumamoto


Earthquake and the Great East Japan Earthquake respectively.
7.0 = 23 log E1 + K ........... (1)

9.0 = 23 log E2 + K ........... (2)

(2) - (1): 2 = 23 log E2 - 23 log E1

2 = 23 (log E2 - log E1)


E
2 = 23 log E2 ◀ log M - log N = log M
1 N
E
log E2 = 3
1

E2
E1
= 103
E2 = 1 000 E1
` The energy released in the Great East Japan Earthquake
was 1 000 times the energy released in the Kumamoto
Earthquake.

Instant Drill 24
In an earthquake, the magnitudes of the main shock and the aftershock
were 8.5 and 6.7 respectively. If the energy released in the main shock
was p times the energy released in the aftershock, find the value of p.
(Give the answer correct to 3 significant figures.)
➥ Ex 8E 30

Richter scale 黎克特制 joules 焦耳

E_SrSecMath_4B08.indd 44 22/5/2023 下午2:45


Logarithmic Functions | 8.45

Class Practice 8.5 ✂

Non-foundation
Exam-type 123
1. Among 10 , 9126 and 8129, which one is the least?

2. The population of a town is 200 000 now. If the population increases


at a constant rate of 1.5% every year, at least how many years
later will the population exceed 250 000?
Exam-type
3. The graph in the figure shows the linear relation between x and log3 y. log3 y
x
It is given that y = ab , where a and b are constants. Find the
values of a and b.
(Leave the radical sign ‘ ’ in the answers if necessary.) 2

x
–4 0

Exam-type
4. The graph in the figure shows the linear relation between log2 x and log2 y

log2 y. Its slope and intercept on the horizontal axis are 1 and -5
5
k
respectively. It is given that y = Cx , where C and k are constants.
Find the values of C and k.
log2 x
–5 0

Exercise 8E
[In this exercise, unless otherwise stated,
(i) give the answers correct to 3 significant figures if necessary,
2
(ii) the relation between the intensity of sound I W/m and its intensity level
D dB is D = 10 log I , where I0 = 10-12 W/m2,
I0
(iii) the relation between the magnitude M and the energy E J released in an
earthquake is M = 2 log E + K, where K is a constant.]
3
Level 1
Exam-type
1. (a) Find the values of log 202300, log 212290 and log 222280.

(b) Among 202300, 212290 and 222280, which one is the greatest? ➥ Example 18

Exam-type
2. (a) Find the values of log 1 234789, log 2 345678 and log 3 456567.

(b) Among 1 234789, 2 345678 and 3 456567, which one is the least?

Exam-type
3. Arrange 600241, 700239 and 800190 in ascending order.

Exam-type
4. Among 101 500
, 103 000 and 109 000
, which one is the best estimate of
Explain 451
767 ? Explain your answer.
8.46 | Chapter 8

2 3
✂ 5. The relation between the surface area A cm and the volume V cm of a
sphere can be represented by the following formula:
Non-foundation

n
A = 4.836V , where n is a constant.
2
It is given that the surface area and the volume of a sphere are 1 000 cm
and 2 974 cm3 respectively. Find the value of n by using logarithms.
Exam-type
6. The graph in the figure shows the linear relation between x and log y. log y
The slope and the intercept on the vertical axis of the graph are 1 and
3 respectively. It is given that y = abx, where a and b are constants.
(a) Express log y in terms of a, b and x.
3
(b) Hence, find the values of a and b. ➥ Example 20
x
0

Exam-type
7. The graph in the figure shows the linear relation between x and log4 y. It log4 y
is given that y = abx, where a and b are constants. x
0 3
(a) Express log4 y in terms of a, b and x. 2

(b) Hence, find the values of a and b.


(c) Find the value of y when x = 3. –3

Exam-type
8. The graph in the figure shows the linear relation between log2 x and log2 y. log2 y

The slope and the intercept on the horizontal axis of the graph are - 5 log2 x
3 0
–3
and -3 respectively. It is given that y = ax b, where a and b are
constants. Find the values of a and b. ➥ Example 21

Exam-type
9. The graph in the figure shows the linear relation between log x and log y. log y

It is given that y = axb, where a and b are constants. Find the values of
a and b. 2
( 3 ,1 )

log x
0

Exam-type x
10. It is given that y = 16(2 ). Denote the graph of log2 y against x by G.
(a) Express log2 y in terms of x.
(b) Find the intercept on the vertical axis and the slope of G.
Exam-type 4
11. It is given that y = 81x . Denote the graph of log3 y against log3 x by H.
(a) Express log3 y in terms of log3 x.
(b) Find the intercept on the horizontal axis and the slope of H.

E_SrSecMath_4B08.indd 46 22/5/2023 下午2:45


Logarithmic Functions | 8.47

Exam-type
12. It is given that y = loga x + 5, where a is a positive constant. If the ✂
graph of y = loga x + 5 passes through (81 , 1), express x in terms of y.

Non-foundation
-4 2
13. If the intensity of a sound produced by a moving car is 10 W/m ,
what is the intensity level of the sound? ➥ Example 23

14. The intensity level of the sound of a crying baby is 68 dB. Find the
intensity of the sound and give the answer in scientific notation.

15. An earthquake occurred in city P. The energy released in the earthquake

is 1012 J. By substituting K = -3.2 into the formula M = 2 log E + K,


3
find the magnitude M of the earthquake.

16. The magnitude M of an earthquake that occurred in city Q is 7.2. By


substituting K = -3.2 into the formula M = 2 log E + K, find the
3
energy E J released in the earthquake and give the answer in scientific
notation.

Level 2
Exam-type -333
17. Arrange 666 , 555-444 and 444-555 in ascending order.
Exam-type -250 -500 -750
18. Among 10 , 10 and 10 , which one is the best estimate of
Explain
213-109? Explain your answer.

19. The value of a machine is $450 000 now. Its value depreciates by 20%
every year. At least how many years later will its value be less than
$100 000? ➥ Example 19

20. Louis deposits $30 000 in a bank at an interest rate of 5% p.a.


compounded yearly. At least how many years later will he receive an
amount more than twice the principal?

21. Mr Chan borrows $10 000 from a bank at an interest rate of 12% p.a.
compounded monthly. For at most how many months can Mr Chan
borrow the money so that the total repayment to the bank will not
exceed $11 000?

E_SrSecMath_4B08.indd 47 22/5/2023 下午2:45


8.48 | Chapter 8

Exam-type
log5 y
✂ 22. The graph in the figure shows the linear relation between log5 x and
log5 y. The intercepts on the horizontal axis and on the vertical axis of
Non-foundation

1
the graph are - 1 and 1 respectively.
2
(a) Find the slope of the graph. log5 x
0
(b) Express y in terms of x. –1
2

Exam-type
23. The graph in the figure shows the linear relation between x and log y. log y

The slope and the intercept on the horizontal axis of the graph are - 1
4
and -2 respectively. Express y in terms of x. x
–2 0

Exam-type
24. It is given that log2 y is a linear function of log2 x. The graph of the
linear function passes through (1 , 5) and (2 , 7). Express y in terms of x. ➥ Example 22
Exam-type
25. It is given that log4 y is a linear function of x. The graph of the linear
function passes through (1 , 2) and (3 , 12). Express y in terms of x.
Exam-type
26. It is given that log2 y is a linear function of log2 x. The intercepts on
the horizontal axis and on the vertical axis of the graph of the linear
36
function are 6 and 3 respectively. If xayb = 2 , where a and b are
constants, find the values of a and b.
-8
27. The intensity of the sound in a bookstore is 10 W/m2.
(a) Find the intensity level of the sound.
(b) If the intensity level of the sound is halved now, find the new
intensity of the sound.

28. The intensity level of a sound is 40 dB. If the intensity level of the
Explain
sound is increased by 25%, is the new intensity of the sound 10 times
the original? Explain your answer.

29. The intensity level of the sound recorded daily on a road is 50 dB. On
a holiday, some people engage illegal motor racing on the road. The
intensity of the sound produced is 106 times as large as that recorded
daily. What is the intensity level of the sound produced in the car race?

30. An earthquake of magnitude 8 occurred in an area. Later, an aftershock


of magnitude 6.5 occurred. How many times the energy released in the
aftershock was the energy released in the first earthquake? ➥ Example 24

31. An earthquake of magnitude 5.4 occurred in area A. Another earthquake


occurred in area B, which released 15 times the energy released in the
earthquake in area A. Find the magnitude of the earthquake in area B,
correct to 1 decimal place.

E_SrSecMath_4B08.indd 48 22/5/2023 下午2:45


Logarithmic Functions | 8.49

Level 3 ✂
Exam-type

Non-foundation
32. It is given that log8 y is a linear function of log2 x. The intercepts on
the horizontal axis and on the vertical axis of the graph of the linear
function are -2 and 4 respectively. Express y in terms of x.

33. In a laboratory, the amount of suspended particulate t hours after the


start of an experiment is N units. A researcher finds that the amount
can be expressed by N = ktr, where k and r are constants. The graph in
the figure shows the linear relation between log 9 t and log27 N.
log27 N

`3 , 2 j
2 3

1
3

log9 t
0

(a) Find the values of k and r.


(b) Find the amount of suspended particulate 5 hours after the start of
the experiment.
(c) The amount of suspended particulate exceeds 7 units p hours after
the start of the experiment. Find the minimum integral value of p.

8.6 History of Logarithm Tables and Slide Rules


Scientists often have to handle operations like multiplication and division,
taking the square or the square root of extremely large numbers. Before
the invention of calculators and computers, they used logarithms to do
such calculations, and the calculation tools include the logarithm table.

Interesting Maths
The Scottish John Napier (1550-1617) invented logarithms
and the logarithm table. Before the invention of calculators,
logarithms were widely used in astronomy, trading, navigation,
engineering and military.

E_SrSecMath_4B08.indd 49 22/5/2023 下午2:45


8. 50 | Chapter 8

✂ A Logarithm Table
Non-foundation

In the common logarithm table (for short, the logarithm table), we can find
the approximate value of the common logarithm of any number between 1 ◀ The approximate value is between
and 10. Below is an extract of the logarithm table. 0 and 1, and is correct to
4 decimal places.

Main Column Difference Column


ADD
0 1 2 3 4 5 6 7 8 9 加
1 2 3 4 5 6 7 8 9

2.0 .3010 3032 3054 3075 3096 3118 3139 3160 3181 3201 2 4 6 8 11 13 15 17 19
2.1 .3222 3243 3263 3284 3304 3324 3345 3365 3385 3404 2 4 6 8 10 12 14 16 18
2.2 .3424 3444 3464 3483 3502 3522 3541 3560 3579 3598 2 4 6 8 10 12 14 15 17
2.3 .3617 3636 3655 3674 3692 3711 3729 3747 3766 3784 2 4 6 7 9 11 13 15 17
2.4 .3802 3820 3838 3856 3874 3892 3909 3927 3945 3962 2 4 5 7 9 11 12 14 16

‘12’ in the Difference Column represents the value 0.001 2.

e.g. (i) log 2.2 . 0.342 4


(ii) log 2.33 . 0.367 4

(iii) log 2.487 . 0.394 5 + 0.001 2 = 0.395 7

For a number M not between 1 and 10, its common logarithm can be found
by the method below.
n n
log M = log ( p # 10 ) = log p + log 10 = log p + n, where p is a number
between 1 and 10, and n is an integer.

e.g. (iv) log 2 130 (v) log 0.213


3
= log (2.13 # 10 ) = log (2.13 # 10-1)
3 -1
= log 2.13 + log 10 = log 2.13 + log 10
. 0.328 4 + 3 . 0.328 4 + (-1)
= 3.328 4 = - 0.671 6

Quick Quiz 8.7

Using the logarithm table, find the values of the following logarithms.
(a) log 2.06 (b) log 2.142
(c) log 22.9 (d) log 0.248
Logarithmic Functions | 8. 51

B Antilogarithm Table ✂

Non-foundation
If log x = y, then x is called the antilogarithm of y. ◀
y
In fact, x = 10 .
When the logarithm of a number is given, we can use an antilogarithm
table to find that number. The approximate value of the antilogarithm of ◀ The approximate value is between
any number between 0 and 1 can be found from the antilogarithm table. 1 and 10, and is correct to
4 significant figures.
Below is an extract of the antilogarithm table.

Main Column Difference Column


ADD
0 1 2 3 4 5 6 7 8 9 加
1 2 3 4 5 6 7 8 9

.50 3162 3170 3177 3184 3192 3199 3206 3214 3221 3228 1 1 2 3 4 4 5 6 7
.51 3236 3243 3251 3258 3266 3273 3281 3289 3296 3304 1 2 2 3 4 5 5 6 7
‘5’ in the Difference Column
.52 3311 3319 3327 3334 3342 3350 3357 3365 3373 3381 1 2 2 3 4 5 5 6 7 represents the value 0.005.
.53 3388 3396 3404 3412 3420 3428 3436 3443 3451 3459 1 2 2 3 4 5 6 6 7
.54 3467 3475 3483 3491 3499 3508 3516 3524 3532 3540 1 2 2 3 4 5 6 6 7

e.g. (i) If log x = 0.513, then x . 3.258 .


(ii) If log x = 0.528 6, then x . 3.373 + 0.005 = 3.378.

For a number M not between 0 and 1, its antilogarithm (for short, antilog)
can be found by the method below. Quick Quiz 8.8
Let M = y + n, where y is a number between 0 and 1, and n is an integer.
M y+n Using the antilogarithm
Then antilog M = 10 = 10 = 10y # 10n = (antilog y) # 10n
table, find the values of
e.g. (iii) antilog 2.531 (iv) antilog (-1.469) the antilogarithms of the
2 following numbers.
= (antilog 0.531) # 10 = (antilog 0.531) # 10-2
. 3.396 # 102 . 3.396 # 10-2 (a) 0.523 (b) 0.541 7
= 339.6 = 0.033 96 (c) 1.518 (d) -0.496

C Slide Rule
The slide rule is another calculation tool designed by applying the concept
of logarithms. We can use the slide rule to do multiplication and division. Interesting Maths
The English mathematician
William Oughtred (1575-1660)
invented the slide rule in 1632.

The accuracy of the readings on the slide rule can only be correct to
3 decimal places, and there are certain limitations and disadvantages.
Students can search more
The slide rules are no longer used after calculators appeared in the 20th information about slide rules on
century. Internet.

antilogarithm 逆對數
8. 52 | Chapter 8

✂ Chapter Summary
Non-foundation

1. Definition of Logarithms Example


Let a 2 0 and a ! 1.
y x = ay loga x = y
(a) If x = a , then y = loga x.
(b) If y = loga x, then x = a .
y 1 = 80 log8 1 = 0
3
(c) log10 x (or log x) represents the common 1 000 = 10 log 1 000 = 3
logarithm of x.
1 = b 1 l2 log 1 1 = 2
Note: loga x is defined only when x 2 0. 9 3 3 9

1 = 6-2 log6 1 = -2
36 36

2. Properties of Logarithms Example


y
(a) loga a = y
(i) log8 97 + log8 97 = log8 ` 97 : 97 j = log8 1 = 0
(b) loga 1 = 0
(c) loga a = 1 (ii) log5 30 - log5 6 = log5 30 = log5 5 = 1
6
(d) loga MN = loga M + loga N 1 1
log3 3 log3 3 2
(iii) = = 2 = 41
log3 9 log3 32 2
(e) loga M = loga M - loga N
N
log 2 log 2 log 2
k
(f) loga M = k loga M, where k is a real number (iv) log4 2 = = = = 1
log 4 log 2
2 2 log 2 2
log b M (formula for change of base)
(g) loga M =
log b a

3. Logarithmic Functions Example


Let a be a constant, where a 2 0 and a ! 1. y = log2 x is a logarithmic function with base 2.
y = loga x is called a logarithmic function with
base a and its domain is all positive real
numbers. The value of y can be any real number.

4. Features of Graphs of Logarithmic


Functions
y y

y = loga x
( a > 1) y = loga x
(0< a <1)
x x
0 1 0 1

(a) The x-intercept of the graph is 1.


(b) The graph does not cut the y-axis. It lies on
the right of the y-axis.
Logarithmic Functions | 8. 53

(c) The graphs of y = log 1a x and y = loga x are Example ✂

Non-foundation
the reflection images of each other with
The figure shows the graphs of y = 4x,
respect to the x-axis.
y = loga x and y = logb x. The graph of
x
(d) The graphs of y = a and y = loga x are the y = loga x is the reflection image of the graph of
reflection images of each other with respect x
y = 4 with respect to the line y = x.
to the line y = x. y
(e) (i) When a 2 1, y = 4x y = x

the graph slopes upwards from left to


y = loga x
right.
x
O P
(ii) When 0 1 a 1 1,
y = logb x
the graph slopes downwards from left
to right.
(a) The coordinates of P = (1 , 0)
(b) a = 4
(c) From the graph,
11b14

5. Exponential Equations and Logarithmic


Equations
Example
Let a 2 0 and a ! 1.
(a) Equations with unknown indices are called 22x + 3 = 2x + 2 5x = 7
x
exponential equations. ` 2x + 3 = x + 2 log 5 = log 7
x y x = -1 x log 5 = log 7
(i) If a = a , then x = y.
log 7
x x x=
(ii) If a = y, then log a = log y. log 5
= 1.2 , cor. to 1 d.p.

(b) Equations with unknowns in logarithms are log3 (x + 1) = 1 log (2x - 1) = log x
called logarithmic equations. ` x+1=3 ` 2x - 1 = x
x=2 x= 1
(i) If loga x = p, then x = ap.
(ii) If loga x = loga q, then x = q.

6. Comparing Values with Logarithms Example


When M 2 N 2 0, log M 2 log N.
log 333100 = 100 log 333 . 252
When log M 2 log N, M 2 N.
log 323110 = 110 log 323 . 276
100 110
a log 333 1 log 323
100 110
` 333 1 323

E_SrSecMath_4B08.indd 53 22/5/2023 下午2:45


8. 54 | Chapter 8

✂ 7. Logarithmic Transformation Example


Non-foundation

We can use logarithmic transformation to find log3 y


the unknown constants in a non-linear function
log3 x
from the graph of a straight line. 0 1

Case 1: –2
x
Transform y = ka (where k 2 1 and a 2 1) to
logm y = logm k + x logm a.
The graph in the figure shows the linear relation
log m y
y log m y = log m k + x log m a between log3 x and log3 y.

y = ka x
Intercept on the vertical axis = -2
slope = log m a

Slope = 0 - ]- 2g
log m k
k
x x 1-0
O O
=2
Case 2:
` log3 y = 2 log3 x - 2
Transform y = bxn (where b 2 1 and n 2 1) to 2
log3 y = log3 x - 2
logm y = logm b + n logm x. 2
log3 y = log3 x - log3 9
log m y
y 2
log m y = log m b + n log m x log3 y = log3 x
9
y = bx n 2

log m b
slope = n y= x
9
x log m x
O O

8. Applications of Logarithms in Real-life


Situation
Logarithms are used in the formula of the
intensity level of sound in decibels and the
formula of the magnitude of earthquake in the
Richter Scale.

E_SrSecMath_4B08.indd 54 22/5/2023 下午2:45


Logarithmic Functions | 8. 55

Checkpoint ✂

Non-foundation
In each of the following, put ‘✓’ in if it is correct, and ‘✗’ if it is wrong.
[Nos. 1−8]

1. log2 0 = 1

2. log3 1 = 3

3. Let a, b and c be positive constants, and a ! 1.


If loga b = c, then ac = b.

4. log4 5 + log4 7 = 0
7 5

5. log7 56 - log7 8 = 1

log 90
6. log3 90 = = 30
log 3

7. 6 556
399
2 5 445403
x+1 3x
8. If 4 = 2 , then x = 2.

9. The figure shows the graphs of y = log3 x, y = loga x and y

y = logb x. They cut the x-axis at P. It is given that the graph of y = log3 x
y = logb x is the reflection image of the graph of y = log3 x
with respect to the x-axis.
y = loga x
(a) The coordinates of P are . x
O P
(b) a ( 2 / 1 ) 3
y = logb x
(c) b =

10. If the graph of y = log 1 x is reflected with respect to the line


5

y = x, the image obtained is the graph of .


3
log x + 3
11. Simplify .
l + log x

12. If a = log 4, express log 25 in terms of a.

x 2x + 1
13. Solve 7 = 6 and give the answer correct to 3 significant
figures.

14. If log4 y = 3 log4 x + 1, express y in terms of x.


8. 56 | Chapter 8

✂ Supplementary Exercise 8
Non-foundation

[In this exercise, unless otherwise stated,


(i) give the answers correct to 3 significant figures if necessary,
2
(ii) the relation between the intensity of sound I W/m and its intensity level
D dB is D = 10 log I , where I0 = 10-12 W/m2,
I0
(iii) the relation between the magnitude M and the energy E J released in an
earthquake is M = 2 log E + K, where K is a constant.]
3
Level 1
1. Convert the following expressions into the form log a x = y.
3
-2 2b
(a) 0.01 = 10 (b) 9 2 = 27 (c) d =c

y
2. Convert the following expressions into the form x = a .

(a) -3 = log 1 (b) log256 16 = 0.5 (c) logm 4 = 3n


1 000

3. Without using a calculator, find the values of the following logarithms.

(b) log 9 1
4
(a) log 1 000 000 (c) log 1 000 (d) log125 5
81

4. Find the value of x in each of the following.

(a) log x = 1 (c) logx 8 = - 1


3
(b) log5 x = 4 (d) logx 5 = 2
3 2

5. Without using a calculator, find the values of the following.


log5 8
(a) log12 6 + log12 24 (b) log 9 - 2 log 30 (c)
log5 16

Simplify the following expressions, where x 2 0 and x ! 1. [Nos. 6–8]


4
log 6 x
6. 7. log x + log 1x 8. (logx 100)(3 log x)
log 6 x

9. If log 5 = k, express the following logarithms in terms of k.

(b) log 1
3
(a) log 125 (c) log 5 (d) log5 100
25

Exam-type
10. If log 2 = p and log 7 = q, express the following logarithms in terms of p and q.
(a) log 14 (b) log 3.5 (c) log4 7 (d) log 1 2
7

11. The two curves in the figure represent the graphs of the logarithmic y
functions y = log 1 x and y = log 1 x .
2 4
(a) Write down the corresponding logarithmic functions for C1 and C2.
(b) C1 and C2 cut the x-axis at a point. Find the coordinates of that
x
point. O
C1
C2
Logarithmic Functions | 8. 57

12. According to the graph given in each of the following figures, sketch ◀ The softcopies of these figures
can be downloaded from the

the graph of the required function in the same figure.

Non-foundation
publisher’s website.
(a) y = log6 x (b) y = log0.8 x

y y
12
y = 0.8 x
2 10 y=x
8
1 6
4
x 2
0 1 2 3 4 x
– 12 – 10 – 8 – 6 –4 – 2 0 2 4 6 8 10 12
–1 y = log 1 x –2
6
–4
–2 –6
–8
– 10
– 12

Solve the following exponential equations. [Nos. 13–15]


x
3x - 1 x -
13. 3 =9 14. 30 = 65.4 15. 25 6 = 800

Solve the following logarithmic equations. [Nos. 16–19]

16. 1 log (1 - x) = 1 17. 2 log 5 = log (x + 1)


3
2

18. log2 (15x - 2) = -log2 7 19. log (2x + 1) + log 1 = 0


5
Exam-type 140
20. Arrange 4 030 , 4 100145 and 4 400144 in ascending order.

21. Anthony deposits a sum of money in a bank. The amount $A that he


can receive after n years is given by the following formula:
A = 2 000 # 1.07n
At least how many years later will Anthony receive the amount more
than $2 800?
Exam-type
22. The graph in the figure shows the linear relation between x and log y. log y
The slope and the intercept on the vertical axis of the graph are both x
0
-3. It is given that y = abx, where a and b are constants.
(a) Express log y in terms of a, b and x.
–3
(b) Find the values of a and b.

E_SrSecMath_4B08.indd 57 22/5/2023 下午2:46


8. 58 | Chapter 8

Exam-type
✂ 23. The graph in the figure shows the linear relation between x and log3 y. log3 y
It is given that y = abx, where a and b are constants.
Non-foundation

(– 1 ,2 )
(a) Express log3 y in terms of a, b and x.
(b) Find the values of a and b.
x
0 3
5

x
24. The figure shows the graph of y = ab , where a and b are constants. y

(a) Find the value of a. Hence, express log6 y in terms of b and x.


y = abx
(b) Sketch a graph of log6 y against x.
1
6

x
0

Exam-type
25. The graph in the figure shows the linear relation between log9 x and log9 y
log9 y. It is given that y = axb, where a and b are constants. 3
2
(a) Find the values of a and b.
(b) Find the value of y when x = 16.
log9 x
–1 0

26. The intensity level of the sound at a concert is 92 dB. What is the
intensity of the sound at the concert?
(Give the answer correct to 2 significant figures and in scientific notation.)

27. Define the magnitude M of an earthquake as M = 2 log E - 3.2, where


Explain
3
E J is the energy released in the earthquake. Two earthquakes occurred
in an area. The magnitude of the first earthquake was 6.4. The energy
18
released in the second earthquake was 10 J. Which earthquake
released a greater amount of energy? Explain your answer.

Level 2
Without using a calculator, find the values of the following. [Nos. 28–29]
4
28. (a) log8 23 (b) log2 64

(c) log 0.13 (d) log 1 4


2

1 log 81 - log 12 - log 3 4 log3 6


29. (a) 2 2 2 (b) (log7 4)(log2 7 ) (c) log3 4 -
2 log12 6

Simplify the following expressions, where x 2 0, y 2 0 and x ! 1, y ! 1. [Nos. 30–32]


2
log2 x - log2 x
4 log (xy) - log xy 2
(log x)(log x y )
30. 31. 32.
log2 1x log 1y
2
log y

E_SrSecMath_4B08.indd 58 22/5/2023 下午2:46


Logarithmic Functions | 8. 59

Exam-type
33. If log 2 = x and log 3 = y, express the values of the following logarithms in terms of x and y. ✂

Non-foundation
(a) log 1.5 (b) log 15 (c) log24 1 (d) log 5
6 27

34. In each of the following, express x in terms of y.

(a) log5 (3x - 4y) = 2 (b) log3 x + log3 y = 2 (c) 1 log x3 = 1 + log (5y)
4
Exam-type
35. In the figure, the graph of y = b 1 l is the reflection image of the graph
x y
3 y=x
of y = loga x with respect to the line y = x. The graphs of y = loga x and
y = logb x cut the x-axis at a point P.
(a) Find the value of a. y = ` 13 j x
x
(b) Find the coordinates of P. O P
y = logb x
y = loga x
(c) Find the range of possible values of b.

Solve the following exponential equations. [Nos. 36–39]


x 2x + 1 x x-1
36. 5 :25 =1 37. 2 + 2 - 48 = 0
3x + 1 x+1
38. 2 :5
x
= 12 39. 7 :6x - 2 = 9x - 1

Solve the following logarithmic equations. [Nos. 40–43]


40. log x + log100 x = 2 41. log16 x = log2 x + 3

42. log2 (3x - 1) - 1 = log2 x 43. log7 (2x - 1) = log7 (8x - 10) + log 7 1
7
Exam-type x x-1
44. (a) Solve 10 - 10 = 9.
x-1
(b) Solve log (10 + 9) = x.
Exam-type x+2
45. (a) Simplify 2 - 2 x.
x
(b) Solve log2 (2 + 12) = x + 2.

46. The value of an antique watch is $13 000 now. Its value increases by
4% every year. At least how many years later will the value of the
watch be more than $20 000?

47. A biologist investigates the number of fish in a lake. The number P


of fish in the lake n years after the start of the investigation can be
represented by the following formula:
P = 1 000(1 + k)n, where k is a positive constant.
It is given that the number of fish is 1 100 when n = 1.
(a) Find the value of k.
(b) At least how many years later will the number of fish be more
than two times the initial number of fish when the lake is first
investigated?

E_SrSecMath_4B08.indd 59 22/5/2023 下午2:46


8. 60 | Chapter 8

✂ 48. Given that for every metre that a diver goes down, the light intensity
decreases by 2.5%. How many metres should a diver go down such that
Non-foundation

(a) the light intensity is one fourth of that at the water surface?
(b) the light intensity is 1% of that at the water surface?
(Give the answers correct to the nearest integer.)

49. The graph in the figure shows the linear relation between x and log3 y. log3 y
It is given that y = mnx, where m and n are constants. Jasmine claims
Explain
x
that both m and n are less than 1. Do you agree? Explain your answer. O

Exam-type
50. It is given that log 4 y is a linear function of log 8 x. The slope and
intercept on the vertical axis of the graph of log4 y against log8 x are
- 43 and 25 respectively.

(a) Express y in terms of x.


(b) (i) Find the value of y when x = 16.
(ii) Find the value of x when y = 64.
Exam-type
51. It is given that log 2 y is a linear function of log 2 x. The intercepts
on the horizontal axis and on the vertical axis of the graph of log2 y
against log2 x are 1 and 3 respectively. If xayb = 26, where a and b are
constants, find the values of a and b.

52. A scientist defined Scale P and Scale Q to represent the magnitude of


Explain
Scale Formula
an explosion as shown in the table. M and N are the magnitudes of an
explosion on Scale P and Scale Q respectively, while E is the energy P M = log3 E
released by the explosion. The magnitude of the explosion in experiment Q N = log7 E
X is 6.5 on Scale P. The magnitude of the explosion in experiment Y is
4.5 on Scale Q. In which experiment will more energy be released by the
explosion? Explain your answer.

53. (a) Alan records the intensity level of the sound in a classroom to be
65 dB. The intensity level of the sound recorded on a street near
his school is 90 dB. How many times the intensity of the sound in
the classroom is that on the street?
(b) Alan knows that the intensity of the sound from a building site is
70 times that from the street mentioned in (a). By how many dB is
the intensity level of the sound from the building site more than
that from the street?
(Give the answers correct to the nearest integer.)

E_SrSecMath_4B08.indd 60 22/5/2023 下午2:46


Logarithmic Functions | 8. 61

54. The table shows the information about the three earthquakes that
Date Magnitude ✂
occurred in Mexico from 1985-1986.

Non-foundation
19/9/1985 8.l
(a) How many times the energy released in the second earthquake was
that in the first one? 21/9/1985 7.5
(b) How many times the energy released in the third earthquake was 30/4/1986 7.0
that in the first one?
(Give the answers correct to the nearest integer.)

Level 3
xy
55. It is given that log3 x = log3 y = log3 12 = k and z = 81, where x, y,
2

z
z and k are positive numbers.
(a) Find the value of k.

(b) Find the values of x, y and z.

56. (a) Rewrite 2 - log 25 in the form of log k, where k is a constant.


x-1
(b) Solve x(2 - log 25) = log (4 + 12).

57. Without using a calculator, show that 3 1 log2 3 1 5 .


2 3

58. The graph in the figure shows the linear relation between log 36 x and log6 y

log6 y. The inclination of the graph is 45c . It is given that y = kxp,


where k and p are constants.
(a) Show that k 2 1. 45°
log36 x
O
(b) Find the value of p.

Cross-topic Questions
2
59. If a and b are the two roots of the equation x - 3x + 1 = 0 (where
a 2 b), find the values of the following expressions.
2a
(c) log3 3 b
a 2b
(a) log a + log b (b) 4 :2
9
(Leave the radical sign ‘ ’ in the answers if necessary.)

60. The figure shows the graphs of C: y = log2 x and L: y = x + 2. C cuts y


L: y = x + 2
the x-axis at point A. B and D are points on L and C respectively such
that AB = BD and AB = the x-axis. B
D
(a) Find the coordinates of A, B and D.
C: y = log2 x
(b) L1 is the perpendicular bisector of the line segment joining A and D. x
O A
L1 cuts the y-axis at point E.
(i) Find the equation of L1.
Explain (ii) Is 3ODE an isosceles triangle? Explain your answer.

E_SrSecMath_4B08.indd 61 22/5/2023 下午2:46


8. 62 | Chapter 8

✂ Multiple-choice Questions
Non-foundation

y-z Exam-type
61. If x = 100 , then z = 69. The figure shows the graphs of y = logp x and
y x y = logq x, where p and q are positive
A. 100 - x. B. y - 100 .
constants.
log x y
C. y - 2 log x. D. y - 2
.

62. If x and y are positive numbers with x ! 1, then


log x (x2y) = O
x
y = log p x
A. 2 (log x)(log y). B. 2 log x + log y.
log y 2 log y y = log q x
C. 2 + . D. .
log x log x
Which of the following must be true?
63. If 1 1 r 1 s, which of the following must be
greater than 1? A. 1 1 p 1 q B. 1 1 q 1 p
C. 0 1 p 1 q 1 1 D. 0 1 q 1 p 1 1
A. logr s B. logs r
C. log r + log s D. (log r)(log s) Exam-type
70. The figure shows the graphs of y = log 4 x
Exam-type and y = log 1 x. They intersect at point M. A
64. If 10 = 2 and 10 = 7, then log 40 =
p q
3
7
vertical line L cuts the graph of y = log4 x, the
A. 2p + 1 - q. B. 2p + 10 - q. graph of y = log 1 x and the x-axis at A, B and
3
2 2p + 1
C. p + 10 - q. D. q . C respectively. Which of the following must
be true?
65. If 2 - log3 (x - K) = 0, then x = I. Coordinates of M = (1 , 0)
A. 100 + K. B. 9 + K. II. AC 1 BC y
L
C. 10
2+K
. D. 3
2+K
. III. AC = log4 3
BC
y = log4 x
A. I and II only A
2
66. If log5 x = 2 log5 3 + log5 4, then x = x
B. I and III only O M C
A. 6. B. 6 or -6. C. II and III only B
C. 36. D. 36 or -36. y = log 1 x
D. I, II and III 3

Exam-type
Exam-type
67. Which of the following is the least? 71. The graph in the figure shows the linear
765 654
A. 432 B. 543 relation between log2 x and y . If x = 16,
543 432
C. 654 D. 765 then y =

Exam-type A. 1. y
68. Let a, b and c be positive constants. On the 4
same rectangular coordinate system, the graph B. 1.
of y = a - logb x and the graph of y = logc x C. 4. 1
slope = 1
4
cut the x-axis at the points P and Q D. 16.
respectively. Denote the origin by O. Find log2 x
0
OP : OQ.
a a
A. b : 1 B. c : 1
a a
C. b : c D. c : c

E_SrSecMath_4B08.indd 62 22/5/2023 下午2:46


Logarithmic Functions | 8. 63

Exam-type Exam-type x
72. The graph in the figure shows the linear 74. The figure shows the graph of y = ab , where ✂
relation between x and log5 y. If y = abx, a and b are constants.

Non-foundation
then b = y

1 . log5 y
A. y = ab x
125
B. 1 .
25 x
0 1.5 6
C. 25.
x
D. 125. 0

–3
Which of the following may represent the
Exam-type relation between x and log5 y?
73. In the figure, the straight line L shows the
A. log5 y B. log5 y
linear relation between log2 x and log2 y. It is
given that L passes through (1 , 12) and
(5 , 44). If y = axb, then a =
A. 8. log2 y x x
L O O
B. 16.
log5 y log5 y
C. 24. C. D.
D. 32. x x
O O

log2 x
O

Competition Corner
1. Australian Mathematics Competition 2018 − Senior Division
x
In the equation g 256 = 2(8 ) the value of x is
60
A. -17 B. -19 C. -21 D. -23 E. 16

2. University of Waterloo − Fermat Contest 2017


If m and n are positive integers with n 2 1 such that mn = 225 # 340, then m + n is
A. 209 962 B. 1 954 C. 209 957 D. 6 598 E. 1 049 760

Solving Strategy: 1 1
1
x 8
g 256 = 2(8 ) as (g ((2 ) 2 ) 2 g) .
2
1. Rewrite
60
2. Consider the product of prime factors of m.
8. 64 | Chapter 8

Conventional Question Tutor Exam Video


(for DSE Level 4+)
Question 1
Denote the graph of y = a + logb x by G, where a and b are constants. It is given that G passes through (64 , 0)
and (1 024 , 2). Express x in terms of y. (4 marks)

Star Tips
① Substitute the coordinates of the two points that G passes through into the equation of G to set up simultaneous equations.
② Use the properties of indices or logarithms to solve the simultaneous equations and find the values of a and b. Then, express
x in terms of y by the definition of logarithms.

S este Sol tion Alternative

a G passes through (64 , 0) and (1 024 , 2). y = a + logb x


y - a = logb x
0 = a + logb 64 ........... (1)
>;7;?

`
2 = a + logb 1 024 ...... (2)
✓ 1 for either one x=b
y-a

a G passes through (64 , 0) and (1 024 , 2).


(2) - (1): 2 = logb 1 024 - logb 64 ✓ 1 for solving
simultaneous 64 = b0 - a .......... (3)
>;7;?

2 = logb 1 024 ` ✓ 1 for either one


64 equations 1 024 = b
2-a
.......... (4)
2 = logb 16 Subtraction is used when
(4) 1 024 2-a

` b = 16
2 logarithms are involved.
(3)
:
64
= b -a
b
✓ 1 for solving simultaneous
equations
b = 4 or -4 (rejected) 16 = b2
Division is used
Substitute b = 4 into (1). b = 4 or -4 (rejected) when indices
0 = a + log4 64 are involved.
Substitute b = 4 into (3).
0=a+3 -a
64 = 4
a = -3 ✓
1 for correct values of a and b
43 = 4-a
` y = -3 + log4 x
y + 3 = log4 x
a = -3 ✓
1 for correct values of a and b

x=4
y+3

1 for correct answer ` x=4
y+3
✓ 1 for correct answer

am Drill 1
Denote the graph of y = a + logb x by G, where a and b are constants. It is given that G passes through (216 , 4)
and the x-intercept of G is 1 . Express x in terms of y. (4 marks)
6

E_SrSecMath_4B08.indd 64 22/5/2023 下午2:46


Logarithmic Functions | 8. 65

MC Question Tutor

Question 2
It is given that log25 y is a linear function of log5 x. The intercepts on the horizontal axis and on the vertical
axis of the graph of the linear function are 8 and 2 respectively. Which of the following must be true?
2 8 2 8
A. xy = 5 B. x y = 5
8 16 8 16
C. xy = 5 D. x y = 5

S este Sol tion

log25 y - 2 = 0 - 2 (log5 x - 0) ◀ Two-point form of straight lines Star Tips


8-0
We can find the relation between x and y by
log25 y - 2 = - 1 log5 x
4 the following steps:
① Find the equation of the graph (a straight
log25 y = - 1 log5 x + 2 line).
4
log5 y ② Find the relation between x and y by the
= - 1 log5 x + 2 definition and the properties of logarithms.
log5 25 4
2 log5 y = -log5 x + 8
2 -1 8
log5 y = log5 x + log5 5
log5 y2 = log5 (58 x-1)
2 8 -1
y =5 x
2 8
xy = 5 Exam Video
(for DSE Level 5**)
` The answer is A.

am Drill 2
It is given that log8 y is a linear function of log2 x. The graph of the linear function passes through (5 , 1)
and (10 , 0). Which of the following must be true?
5 6 5 6
A. xy = 2 B. x y = 2
3 5 30 5 3 30
C. x y = 2 D. x y = 2

Exam Information
Exam-type Questions

1. The graph in the figure shows the linear relation between log3 x and log9 y

log9 y. The slope of the graph is - 21 and the intercept on the horizontal
b
axis is 4. Express the relation between x and y in the form y = ax ,
where a and b are constants.
log3 x
0 4

E_SrSecMath_4B08.indd 65 22/5/2023 下午2:46


8. 66 | Chapter 8

2. Denote the graph of y = a + logb x by G, where a and b are constants. It is given that G passes through
(1 , -2) and (8 , -1). Express x in terms of y.

3. Which of the following is the best estimation of 6. The graph in the figure shows the linear relation
9 898300? between log5 x and log5 y. Which of the
A. 101 000 following must be true?
B. 10
1 500 2 log5 y
y 12
2 000
A. 3
=5
C. 10 x
D. 102 500 y
3
12 4
B. 2
=5
x
4. Which of the following may represent the graph 2
of the function y = log 2 x ? C. x = 512
3 log5 x
3 y –6 0
y y
A. B. 3
x = 512
D. 2
y
x
O
7. The graph in the figure shows the linear relation
x
O between x and y . Which of the following
must be true?
y y
C. D.
A. y = 1 x - 3x + 9
2 y
4
B. y = 1 x + 3x + 9
x 2
O 4
x 3
O 2
C. y = 4x - 12x + 9 x
2 0
D. y = 4x + 12x + 9 6
5. The figure shows the graphs of y = logp x and
y = logq x, where p and q are positive constants. 8. It is given that log6 y is a linear function of x.
A vertical line L cuts the x-axis, the graph of The slope and the intercept on the horizontal
y = logp x and the graph of y = logq x at A, B axis of the graph of the linear function are
and C respectively. Which of the following
- 41 and 8 respectively. If y = ab , then a =
x

is/are true?
A. 1. B.
I. p1q y
6
6.
L
II. pq 2 1 C. 6. D. 36.
III. BC = log p
q q
AB C 9. It is given that log4 y is a linear function of
y = logp x
A. I only B log8 x. The graph of the linear function passes
x
B. II only O A through (4 , -1) and (-2 , 2). Which of the
y = logq x
C. I and III only following must be true?
D. II and III only 1 1
A. x2 y = 2 B. x y 2 = 2
1 1
C. x3 y = 4 D. xy3 = 4

E_SrSecMath_4B08.indd 66 22/5/2023 下午2:46


Logarithmic Functions | 8. 67

Public Exam Questions Public Exam


Dynamic Figures
1. DS 2017

Let a and b be constants. Denote the graph of y = a + logb x by G . The


x-intercept of G is 9 and G passes through the point (243 , 3). Express
x in terms of y. (4 marks)

2. DS 201

The graph in the figure shows the linear relation between log4 x and log8 y

log8 y. The slope and the intercept on the horizontal axis of the graph
are - 1 and 3 respectively. Express the relation between x and y in
3
k
the form y = Ax , where A and k are constants. (3 marks)
log4 x
O 3
3. C 200 1

Solve the equation 4x + 1 = 8. (3 marks)

4. H DS 2021 6. H DS 2020
Let a, b and c be positive constants. On the same The graph in the figure shows the linear relation
rectangular coordinate system, the graph of between x3 and y . If x = 2, then y =
y = a + logb x and the graph of y = logc x cut A. 3. y
the x-axis at the points S and T respectively. B. 8.
Denote the origin by O. Find OT : OS. C. 9. 1
a
A. 1:b D. 33. x3
B. 1 : ca –4 O

C. ba : 1
7. DS 201
D. ca : 1
It is given that log9 y is a linear function of
5. DS 2021 log 3 x. The intercepts on the vertical axis
The graph in the figure shows the linear relation and on the horizontal axis of the graph of
between log 5 x and log 5 y. Which of the the linear function are 7 and 8 respectively.
following must be true? Which of the following must be true?

A. xy = 625
2 log 5 y A. x 4y 7 = 356
2
B. x y = 625 2 B. x 7y 4 = 356
y
2 C. x 7y 8 = 356
C. x = 625
D. x 8y 7 = 356
y
D. = 625
x
2 log 5 x
–4 O

E_SrSecMath_4B08.indd 67 22/5/2023 下午2:46


8. 68 | Chapter 8

8. H DS 201 11. DS 201

The figure shows the graph of y = loga x and the Which of the following is the greatest?
graph of y = logb x on the same rectangular A. 124241
coordinate system, where a and b are positive B. 241
214

constants. If a vertical line cuts the graph of C. 412142


y = loga x, the graph of y = logb x and the x-axis D. 421
124

at the points A, B and C respectively, which of


the following is/are true? 12. DS 201
y y
I. a21
II. a2b y = loga x

III. AB = log b A y = logb x


a a
BC B
A. I only 3 y = abx

B. II only x
x O
O C
C. I and III only
D. II and III only The figure above shows the graph of y = abx,
where a and b are constants. Which of the
9. DS 201
following graphs may represent the relation
In the figure, the straight line L shows the between x and log7 y?
relation between log4 x and log4 y. It is given A. log7 y B. log7 y
that L passes through the points (1 , 2) and
(9 , 6). If y = kxa, then k =
1. log4 y
A. L
2 x x
O O
B. 3.
2
C. log7 y D. log7 y
C. 2.
D. 8. O
log4 x x x
O O

10. DS 201

The graph in the figure shows the linear relation


between x and log9 y. If y = abx, then b =
A. -2 log9 y 13. C 200

B. 1 . If 5 = 10a and 7 = 10b, then log 7 =


81 50
x
1. O
C. 4 A. b - a - 1.
2
–2 B. b - a + 1.
D. 3.
b.
C. a

D. b .
a+1

E_SrSecMath_4B08.indd 68 22/5/2023 下午2:46


Logarithmic Functions | 8. 69

Answers
1

Q & A (P. 8.1) 22 1. y= 4


x3 2. y=5
4x - 3

32 times 23 10 W/m
2

24 501
Instant Drill
1 a 5 -6 -4 Quick Quiz 8.1 (P. 8.3)
2 a 1 10 000 1. 4 2. 10
6
3. -2
50.1 0.251 4. 10
-3
5. 2.5 6. 10
d

3 a 2 3 1
4 3 Quick Quiz 8.2 (P. 8.4)
-1 e f
3 4
1. 2.28 2. 0.556 3. -0.569
4 a 8 log x 2 log x 6
5 a a+b 2a - b Quick Quiz 8.3 (P. 8.6)
2a + b + 1 b-a-1 1. ✗ 2. ✓ 3. ✗ 4. ✓
6 a 1 2 -4 5. ✓ 6. ✗ 7. ✗ 8. ✓

0 e -1 f 1
3 Class Practice 8.1 (P. 8.9)
7 a 1 2
3 1. 0.1 2. 2 3. -1
8 a 1 2
4. 2 5. -log x 6. 1
2
3 1
2 7 a a+b b-a a+b+1
9 a 6 2
Exercise 8A (P. 8.10)
10 y = x
11 a (i) no (ii) yes
1. a 1.38 3.48 -1.30

- log1 a 2. log 100 000 = 5 3. log 1 = -2


100

1
4. log 0.2 = n 5. 1 000 = 103
12 a -1
3 6. 0.000 01 = 10
-5
7. A = 100.9
5 3 a 7 -1 -3
2
13 a 2 1 9. 10 10. 6.31 11. 0.087 1 12. 3

14 a 1.36 -1.89 13. 0 14. -1 15. 2 16. 2

15 a 0.613 -1.30 -4.86 17. 1 18. 3 19. 2 20. 0


4
16 a 100 1 21. log x 22. 3 23. 1
4
2 a 2a -a a
3 3 2

17 a 1 no real solutions a+1 e 1-a


27
2 a m-n m+n
18 1. 312132
2m + n 2n - 3m
2. 5013 000
26. 2 27. 4 28. 2 29. 1
19 1. 26 months 2. 9 years 5 2
20 a log y = log a + x log b 30. 1 31. 2 32. 1 33. 2

a = 1 , b = 5.62 34. 1 35. 1 36. 1 37. - 3


10 5 2 2 2

21 a = 8, b = 3 38. 4 39. 1 40. 1 41. 3


2 3 4 2
8.70 | Chapter 8

42. 3 43. 2 44. 1 45. 2 39. 1 40. 0 41. 3 42. 6


3 6 2 2
a 3p + q p+q+1 43. 1 44. 1
2
] - pg
p 1 3q
3
- 2q 2
1
2
45. y = x 2 46. y = 64x
7 a 2a + b 3a + b 2
47. y = x 48. y = x
a 1 - b - 4a 81
2
+b
- ]a + 2bg
a a+b
a 1+a+b a- b 2
2
0 a 1 - ba 3a
1 ]a
2a + b
2 3
- bg 8b
p q 2
49. 2q - p 50. 1 × 10
10 51. q 52. p +2 53. y = x 54. 2 - 4r

Quick Quiz 8.4 (P. 8.12) Quick Quiz 8.6 (P. 8.24)

1. log2 16 = 4 1. ✓ 2. ✗ 3. ✓ 4. ✗
2. log 6 36 = 2

3. log5 1 = -2 4. log 12 = m Class Practice 8.3 (P. 8.28)


25
5. 1 = 7
0
6. 81 = 9 2
1 a y = log1.3 x y = 0.3x
1 = 10-1 y = 1.3x y = log0.3 x
7. 8. n = 21.5
10
2 a greater than 6 1
6
Quick Quiz 8.5 (P. 8.13)
a 1.585 0.921 -3.585 Exercise 8C (P. 8.28)
1. a y = log 1 x y = log4 x
Class Practice 8.2 (P. 8.17) 7

y = log5 x y = log 2 x
3
1. -2 2. 16
2 a y = log 7 x y = log0.2 x
a -2 0
y = log 1 x y = log 3 x
a 6 1 6

y = a 31 k
4 x x
1 a y=2
5. y = x 4
y = 0.8x y = ] 5 gx
Exercise 8B (P. 8.18)
a C1: y = log3 x , C2: y = log0.5 x (1 , 0)
1 a 2.32 0.936 -1.29 x
5. C3 : y = 4 , C4 : y = log x
2. 3 3. 5 4. 0 5. -1
a 5 1
6. -4 7. 1 8. 64 9. 16 5
3 9
9. a 3 a2b2 1
3
10. 4 11. 1 12. 1 13. 2
7 10 a (1 , 0)
14. 1 15. 2 16. -1 17. 3 1 1 1n11
(i) (ii)
8 8
18. 1 19. 3 20. 4 21. 2
4 2 11 a (i) yes (ii) no
1 x
22. 2 23. 1 24. 1 a y=4 (16 , 2)
2
14. 0.7
2 a a+1 a-2 2a

26. a 2a + b a+b b
a Class Practice 8.4 (P. 8.35)
2

27. - 1 28. 3 29. - 1 30. 1 1. 0 2. 1 3. 1


2 4 2 2

31. 2 32. 2 33. -1 34. 4 4. 3.32 5. 2.09 6. -1.91


1
35. 6 36. 1 37. 1 38. 1 7.
3
8. 8 9. no real solutions
7 2 3

E_SrSecMath_4B08.indd 70 22/5/2023 下午2:46


Logarithmic Functions | 8.71

Exercise 8D (P. 8.36)


-6
13. 80 dB 14. 6.31 × 10 W/m2
15
15. 4.8 16. 3.98 × 10 J
1. 1 2. 1 3. 6 4. 4
2 17. 444
-555
1 555 -444
1 666 -333

5. 2 6. - 2 7. - 1 8. 1 18. 10
-250
19. 7 years 20. 15 years 21. 9 months
3 2 2
9. 2.58 10. 0.183 11. 6.68 12. 2.26 22 a 2
2
13. 1.04 14. 0.319 15. 5 16. 3 y = 5x
1 1
- x- 2
17. -1 18. 82 19. -19 20. 2 23. y = 10 4 2 24. y = 8x
5x - 3
21. 16 22. 12 23. 9 24. 1 25. y = 4 26. a = 6, b = 12
11 -10 2
27. a 40 dB 10 W/m
25. 2 26. 6 27. 2 28. 12
3 28. yes 29. 110 dB 30. 178 times 31. 6.2
6
29. 4 30. 3 31. 2 32. 2 32. y = 4 096x
33. 5.91 34. 0.361 35. 0.612 36. -3.44
a k = 3, r = 1
3
37. 0.338 38. 2.72 39. 0.01 40. 9
5.13 units 13
41. 64 42. 1 43. 1
25 3
Quick Quiz 8.7 (P. 8.50)
44. no real solutions 45. no real solutions
a 0.313 9 0.330 8
46. 3 47. x = -2, y = 12
r 1.359 8 -0.605 5
48. 4
Quick Quiz 8.8 (P. 8.51)
Class Practice 8.5 (P. 8.45)
a 3.334 3.481
129
1. 8 2. 15 years 32.96 0.319 2
3. a = 9 , b = 3 4. C = 2 , k = 1
5 Checkpoint (P. 8.55)
Exercise 8E (P. 8.45) 1. ✗ 2. ✗ 3. ✓ 4. ✓
300 290
1 a log 202 = 692, log 212 = 675, 5. ✓ 6. ✗ 7. ✓ 8. ✓
log 222280 = 657 a (1 , 0) 2 1
300 3
202
10. y = a 1 k
x

2 a log 1 234789 = 2 440, log 2 345678 = 2 280, 5

log 3 456 567


= 2 010 11. 3 12. 2 - a
3
3 456567 13. -1.09 14. y = 4x
190
3. 800 1 600241 1 700239
Supplementary Exercise 8 (P. 8.56)
1 500
4. 10 5. 0.667
a log y = log a + x log b
1 a log 0.01 = -2 log 9 27 = 3
2
a = 1 000, b = 10 log d c = 2b
7 a log4 y = log4 a + x log4 b 2 a 1 = 10-3 16 = 2560.5
1 000
a = 1 , b = 16 64
64 4 = m3n
8. a = 1 , b = - 5 9. a = 100, b = - 1
32 3 3 a 6 -2 3 1
4 3
10 a log2 y = 4 + x
a 2.15 625 1 1.22
intercept on the vertical axis = 4, slope = 1 64
11 a log3 y = 4 + 4 log3 x a 2 -2 3
4
intercept on the horizontal axis = -1, slope = 4
6. 4 7. 0 8. 6
or x = a 1 k
5-y y-5
12. x = 3 k 2
3 a 3k - 2k 3 k

E_SrSecMath_4B08.indd 71 22/5/2023 下午2:46


8.72 | Chapter 8

q p
10 a p+q q-p -q a 8 times 45 times
2p
11 a C1: y = log 1 x , C2: y = log 1 x 55. a 2 x = 9, y = 3, z = 1
4 2 3
(1 , 0) a log 4 2
13. 1 14. 1.23 15. -12.5 58. 1
2
16. -8 17. 24 18. 1 0 64
7 a 2 5
140 145 144
19. 2 20. 4 030 1 4 100 1 4 400 0 a A: (1 , 0), B: (1 , 3), D: (8 , 3)
21. 5 years (i) 7x + 3y - 36 = 0 (ii) no
22 a log y = log a + x log b 61. D 62. C 63. A 64. A
65. B 66. B 67. D 68. A
a= 1 , b= 1
1 000 1 000
69. C 70. D 71. C 72. C
2 a log3 y = log3 a + x log3 b
73. B 74. A
a = 2.28, b = 0.253

2 a a = 1 , log6 y = -1 + x log6 b Competition Corner (P. 8.63)


6
1. B 2. C
2 a a = 27, b = 3 1 728
2
Exam Get-Set-Go — Exam Drill (P. 8.64)
-3
26. 1.6 × 10 W/m2 27. second earthquake
y-1
2 a 1 3 1 x=6
2 2
2 C
- 23 -2
Exam Get-Set-Go — Exam-type Questions (P. 8.65)
2 a -2 1 -1
2 1. y = 81x
-1
2. x = 8
y+2

30. 2 31. 1 32. -2


3. A 4. C 5. C 6. B
a y-x y-x+1 7. A 8. D 9. C

] - x - 3y g
x+y 1 1
- 3x
+y 2 Exam Get-Set-Go — Public Exam Questions (P. 8.67)

x = ]50yg
25 + 4y 4
34. a x= x = 812 3
1. x = 3
y+2
2. y = 8x
-1
2 3. 1
3 y 2
a 1 (1 , 0) 01b1 1 4. C 5. C 6. C 7. B
3 3
36. - 2 37. 5 38. 0.159 8. C 9. D 10. D 11. B
5
12. B 13. A
39. -0.363 40. 100 41. 1
16
42. 1 43. no real solutions
a 1 1
x
a 3 (2 ) 2
46. 11 years
7 a 0.1 8 years
a 55 m 182 m
49. yes
1
-
0 a y = 32x 2

(i) 8 (ii) 1
4
51. a = 6, b = 2
52. experiment Y
a 316 times 18 dB

E_SrSecMath_4B08.indd 72 22/5/2023 下午2:46

You might also like